OB-Gyn Practice Surg Assessment 7th Edition

Download as pdf or txt
Download as pdf or txt
You are on page 1of 78

PROLOG

Gynecology and Surgery


seventh edition
Gynecology and Surgery
seventh edition

Assessment Book
ISBN 978-1-934984-36-9

Copyright 2014 by the American College of Obstetricians and Gynecologists. All rights
reserved. No part of this publication may be reproduced, stored in a retrieval system,
posted on the Internet, or transmitted, in any form or by any means, electronic, mechanical,
photocopying, recording, or otherwise, without the prior written permission of the publisher.

12345/87654

The American College of Obstetricians and Gynecologists


409 12th Street, SW
PO Box 96920
Washington, DC 20090-6920
Contributors

PROLOG Editorial and Advisory Committee

CHAIR MEMBERS
Ronald T. Burkman, Jr, MD Louis Weinstein, MD
Professor of Obstetrics and Gynecology Past Paul A. and Eloise B. Bowers
Tufts University School of Medicine Professor and Chair
Division of General Obstetrics and Department of Obstetrics and
Gynecology Gynecology
Department of Obstetrics and Thomas Jefferson University
Gynecology Philadelphia, Pennsylvania
Baystate Medical Center Linda Van Le, MD
Springfield, Massachusetts Leonard Palumbo Distinguished
Professor
Division of Gynecologic Oncology
Department of Obstetrics and
Gynecology
University of North Carolina School of
Medicine
Chapel Hill, North Carolina

PROLOG Task Force for Gynecology and Surgery, Seventh Edition

COCHAIRS MEMBERS
John F. Greene, MD Leslie R. DeMars, MD
Chief Medical Officer Associate Professor
MidState Medical Center Division Director
Meriden, Connecticut Gynecologic Oncology
Professor of Obstetrics and Geisel School of Medicine
Gynecology Dartmouth-Hitchcock Medical Center
The Frank H. Netter, MD, School of Lebanon, New Hampshire
Medicine Geri D. Hewitt, MD
Quinnipiac University Associate Professor
North Haven, Connecticut Department of Obstetrics and
James M. Shwayder, MD, JD Gynecology
Professor and Chair Ohio State University Medical Center
Department of Obstetrics and Columbus, Ohio
Gynecology Fred M. Howard, MD
University of Mississippi Medical Professor Emeritus
Center Department of Obstetrics and
Jackson, Mississippi Gynecology
University of Rochester Medical Center
Rochester, New York

Continued on next page

iii
iv
iv PROLOG

PROLOG Task Force for Gynecology and Surgery, Seventh Edition


(continued)
Mary T. Jacobson, MD Colleen K. Stockdale, MD, MS
Adjunct Clinical Associate Professor Clinical Associate Professor
Department of Obstetrics and Department of Obstetrics and
Gynecology Gynecology
Stanford University School of University of Iowa Carver College of
Medicine Medicine
Stanford, California Iowa City, Iowa
Amy M. Johnson, MD
Residency Program Director Note: Geoffrey D. Bowers, MD, also
Department of Obstetrics and contributed to this unit.
Gynecology
University of Connecticut COLLEGE STAFF
Hartford, Connecticut Sandra A. Carson, MD
Michael R. Milam, MD, MPH Vice President for Education
Gynecologic Oncology Sterling B. Williams, MD, MS, PhD
Norton Cancer Institute Past Vice President for Education
Norton Healthcare Campus Erica Bukevicz, MBA, MS
Louisville, Kentucky Director, Educational Development and
Lisa Rahangdale, MD, MPH Testing
Department of Obstetrics and Division of Education
Gynecology Christopher T. George, MLA
Division of Women’s Primary Editor, PROLOG
Healthcare
University of North Carolina School of
Medicine
Chapel Hill, North Carolina
Khaled Sakhel, MD
Associate Professor
Director of Minimally Invasive
Gynecology and Robotic Surgery
Department of Obstetrics and
Gynecology
Eastern Virginia Medical School
Norfolk, Virginia
PROLOG v

CONFLICT OF INTEREST DISCLOSURE


This PROLOG unit was developed under the direction of the PROLOG Advisory Committee
and the Task Force for Gynecology and Surgery, Seventh Edition. PROLOG is planned
and produced in accordance with the Standards for Enduring Materials of the Accreditation
Council for Continuing Medical Education. Any discussion of unapproved use of products is
clearly cited in the appropriate critique.
Current guidelines state that continuing medical education (CME) providers must ensure
that CME activities are free from the control of any commercial interest. The task force
and advisory committee members declare that neither they nor any business associate nor
any member of their immediate families has material interest, financial interest, or other
relationships with any company manufacturing commercial products relative to the topics
included in this publication or with any provider of commercial services discussed in the
unit except for Khaled Sakhel, MD, who is a consultant for Hologic and a speaker for
Conceptus; James M. Shwayder, MD, who receives royalties from Cook Ob-Gyn for an
SIS catheter and who is a consultant for Philips Healthcare Ultrasound; and Linda Van Le,
MD, who serves on the Advisory Board of Biologics, Inc. All potential conflicts have been
resolved through the American College of Obstetricians and Gynecologist’s mechanism for
resolving potential and real conflicts of interest.
Preface
PROLOG (Personal Review of Learning in Obstetrics and Gynecology) is a voluntary,
strictly confidential, self-evaluation program. PROLOG was developed specifically as a
personal study resource for the practicing obstetrician–gynecologist. It is presented as a self-
assessment mechanism that, with its accompanying performance information, should assist
the physician in designing a personal, self-directed life-long learning program. It may be used
as a valuable study tool, reference guide, and a means of attaining up-to-date information in
the specialty. The content is carefully selected and presented in multiple-choice questions
that are clinically oriented. The questions are designed to stimulate and challenge physicians
in areas of medical care that they confront in their practices or when they work as consultant
obstetrician–gynecologists.
PROLOG also provides the American College of Obstetricians and Gynecologists (the
College) with one mechanism to identify the educational needs of the Fellows. Individual
scores are reported only to the participant; however, cumulative performance data and
evaluation comments obtained for each PROLOG unit help determine the direction for future
educational programs offered by the College.

Process
The PROLOG series offers the most current information available in five areas of the
specialty: obstetrics, gynecology and surgery, reproductive endocrinology and infertility,
gynecologic oncology and critical care, and patient management in the office. A new
PROLOG unit is produced annually, addressing one of those subject areas. Gynecology and
Surgery, Seventh Edition, is the second unit in the seventh 5-year PROLOG series.
Each unit of PROLOG represents the efforts of a special task force of subject experts under
the supervision of an advisory committee. PROLOG sets forth current information as viewed
by recognized authorities in the field of women’s health. This educational resource does not
define a standard of care, nor is it intended to dictate an exclusive course of management.
It presents recognized methods and techniques of clinical practice for consideration by
obstetrician–gynecologists to incorporate in their practices. Variations of practice that take
into account the needs of the individual patient, resources, and the limitations that are special
to the institution or type of practice may be appropriate.
Each unit of PROLOG is presented as a two-part set, with performance information and
cognate credit available to those who choose to submit their answer sheets for confidential
scoring. The first part of the PROLOG set is the Assessment Book, which contains
educational objectives for the unit and multiple-choice questions, and an answer sheet with
return mailing envelope. Participants can work through the book at their own pace, choosing
to use PROLOG as a closed- or open-book assessment. Return of the answer sheet for scoring
is encouraged but voluntary.
The second part of PROLOG is the Critique Book, which reviews the educational
objectives and items set forth in the Assessment Book and contains a discussion, or critique,
of each item. The critique provides the rationale for correct and incorrect options. Current,
accessible references are listed for each item.

Continuing Medical Education Credit


ACCME Accreditation
The American College of Obstetricians and Gynecologists is accredited by the Accreditation
Council for Continuing Medical Education (ACCME) to provide continuing medical
education for physicians.
AMA PRA Category 1 Credit(s)TM
The American College of Obstetricians and Gynecologists designates this enduring material
for a maximum of 25 AMA PRA Category 1 Credits™. Physicians should claim only the
credit commensurate with the extent of their participation in the activity.

vii
viii
viii PROLOG

College Cognate Credit(s)


The American College of Obstetricians and Gynecologists designates this enduring material
for a maximum of 25 Category 1 College Cognate Credits. The College has a reciprocity agree-
ment with the American Medical Association that allows AMA PRA Category 1 Credit(s)™
to be equivalent to College Cognate Credits.
Fellows who submit their answer sheets for scoring will be credited with 25 CME credits.
Participants who return their answer sheets for CME credit will receive a Performance Report
that provides a comparison of their scores with the scores of a sample group of physicians
who have taken the unit as an examination. An individual may request credit only once for
each unit. Please allow 4–6 weeks to process answer sheets.
Credit for PROLOG Gynecology and Surgery, Seventh Edition, is initially available
through December 2016. During that year, the unit will be reevaluated. If the content remains
current, credit is extended for an additional 3 years, with credit for the unit automatically
withdrawn after December 2019.

Conclusion
PROLOG was developed specifically as a personal study resource for the practicing
obstetrician–gynecologist. It is presented as a self-assessment mechanism that, with its
accompanying performance information, should assist the physician in designing a personal,
self-directed learning program. The many quality resources developed by the College,
as detailed each year in the College’s Publications and Educational Materials Catalog
are available to help fulfill the educational interests and needs that have been identified.
PROLOG is not intended as a substitute for the certification or recertification programs of
the American Board of Obstetrics and Gynecology.

PROLOG CME SCHEDULE


Reproductive Endocrinology and Infertility, Credit through 2015
Sixth Edition
Gynecologic Oncology and Critical Care, Credit through 2016
Sixth Edition
Patient Management in the Office, Reevaluated in 2014–
Sixth Edition Credit through 2017
Obstetrics, Seventh Edition Reevaluated in 2015–
Credit through 2018
Gynecology and Surgery, Seventh Edition Reevaluated in 2016–
Credit through 2019
PROLOG Objectives

PROLOG is a voluntary, strictly confidential, personal continuing education resource that is


designed to be both stimulating and enjoyable. By participating in PROLOG, obstetrician–
gynecologists will be able to do the following:
• Review and update clinical knowledge.
• Recognize areas of knowledge and practice in which they excel, be stimulated to
explore other areas of the specialty, and identify areas requiring further study.
• Plan continuing education activities in light of identified strengths and deficiencies.
• Compare and relate present knowledge and skills with those of other participants.
• Obtain continuing medical education credit, if desired.
• Have complete personal control of the setting and of the pace of the experience.

The obstetrician–gynecologist who completes Gynecology and Surgery, Seventh Edition,


will be able to:
• Establish a differential diagnosis and screen patients with appropriate diagnostic
tests for specific gynecologic conditions.
• Determine the appropriate medical management for specific gynecologic conditions in
adolescents and adult women.
• Identify appropriate surgical interventions for various gynecologic conditions and
strategies to prevent and treat surgical complications.
• Apply concepts of anatomy, genetics, pathophysiology, and epidemiology to the
understanding of diseases that affect women.
• Counsel women regarding treatment options and adjustment to crises that may alter their
lifestyles.
• Apply professional medical ethics and the understanding of medical–legal issues relative
to the practice of gynecology.
Gynecology and Surgery, Seventh Edition, includes the following topics:

SCREENING AND DIAGNOSIS


Abnormal cervical cytology results
Abnormal uterine bleeding in an adolescent patient
Atypical glandular cells Pap test result
Cervical cancer screening and diagnosis
Chronic pelvic pain
Complications of uterine artery embolization
Eating disorders and irregular menstruation
Dysmenorrhea in an adolescent patient
Frequent urination
Intraoperative hypotension
Levonorgestrel intrauterine device with missing strings
Ovarian cancer screening
Ovarian mass in an adolescent patient
Pap test result in a patient older than 40 years that shows endometrial cells
Pediatric vaginitis
Perioperative cardiac evaluation
Persistent vulvar pain
Recurrent urinary incontinence
Recurrent urinary tract infection
Recurrent yeast infections

ix
x
x PROLOG

Screening for vaginal intraepithelial neoplasia


Severe dyspareunia
Sexual dysfunction in menopausal women
Tumor markers
Thrombophilias and contraception
Vaginal cancer screening after hysterectomy
Vesicovaginal fistula

MEDICAL MANAGEMENT
A patient with ductal carcinoma in situ
A patient with sexual partner who has genital herpes
Abnormal uterine bleeding in an adolescent patient
Abnormal uterine bleeding in a reproductive-aged woman
Adnexal mass in a postmenopausal woman
Adverse effects of pharmacologic therapy for overactive bladder
Appropriate treatment of sexually transmitted diseases
Asymptomatic myomas
Beta-blocker therapy in perioperative care
Breast abscess
Clinical depression
Complex endometrial hyperplasia
Contraception in a patient with gastric bypass
Contraception in the morbidly obese patient
Contraceptive options for a woman with diabetes mellitus and a seizure disorder
Depot medroxyprogesterone acetate and bone loss
Dysmenorrhea
Ectopic pregnancy
Endometriosis
Evaluation of urinary incontinence
Extended-cycle combined oral contraceptives
First-trimester pregnancy loss
Fibrocystic breast disease
Follow-up after loop electrosurgical procedure
Heavy menstrual bleeding
Heavy menstrual bleeding in a patient with a low platelet count
Hormone therapy after hysterectomy for endometriosis
Hormone therapy after hysterectomy
Human immunodeficiency virus exposure and prophylaxis
Hydrosalpinx
Hysteroscopic complications
Hysteroscopic sterilization
Immunization
Lichen sclerosus
Mature cystic teratoma with recurrence
Metformin hydrochloride and polycystic ovary syndrome
Molar pregnancy
Nonhormonal therapy for menopause
Obesity
Office hysteroscopy
Outpatient management of pelvic inflammatory disease
Ovarian cancer in BRCA 1 and BRCA 2 carriers
Ovarian mass in an older reproductive-aged woman
Ovarian remnant syndrome
PROLOG xi
Gynecology and Surgery xi

Perioperative care of a patient with diabetes mellitus


Postmenopausal bleeding
Postmenopausal bleeding with hormone therapy
Premalignant and in situ breast disease
Recalcitrant condyloma
Recommendations to prevent osteoporosis
Recurrent bacterial vaginosis
Recurrent breast mass
Recurrent genital herpes
Recurrent urinary tract infection
Recurrent vaginal yeast infection
Sexual dysfunction
Single-rod implantable contraception
Squamous dysplasia of the cervix
Unplanned pregnancy
Urinary retention after an antiincontinence procedure
Use of levonorgestrel intrauterine device for heavy menstrual bleeding
Vaginal intraepithelial neoplasia
Wound management

PHYSIOLOGY
Anterior abdominal wall anatomy and surgical complications
Vascular supply of the pelvis

SURGICAL MANAGEMENT
Adenocarcinoma in situ with positive margins
Alternatives to hysterectomy for myomas
Anterior abdominal wall anatomy and surgical complications
An ovarian tumor of low malignant potential
Bradycardia during minor gynecologic surgery
Endometrial ablation
Ethical management of unexpected surgical findings
Incomplete abortion in the second trimester
Indications for risk-reducing surgery
Intraoperative hypotension
Oliguria in the elderly patient
Ovarian torsion
Ovary preservation or removal at the time of hysterectomy
Pelvic floor anatomy
Pelvic organ prolapse
Perioperative anticoagulants
Perioperative pulmonary complications
Postoperative delirium in an elderly patient
Postoperative intestinal obstruction
Postoperative management of nerve injury
Postoperative oliguria
Postoperative pelvic abscess
Prophylactic antibiotics for abdominal surgery
Surgery for uterovaginal prolapse
Use of prophylactic antibiotics in surgery for patients allergic to penicillin
Uterosacral ligament suspension for vaginal prolapse
Vulvar vestibulitis
Wound dehiscence
xii
xii PROLOG

EPIDEMIOLOGY AND BIOSTATISTICS


Atypical endometrial hyperplasia and endometrial cancer
Carcinoma risk reduction
Familial cancer syndromes
Risk of acquiring human immunodeficiency virus after sexual assault
Risk reduction factors for ovarian cancer

COUNSELING
Atypical endometrial hyperplasia and endometrial cancer
Complications of embolization
Contraception in a patient with gastric bypass
Contraception in the morbidly obese patient
Eating disorders and irregular menstruation
Emergency contraception
End-of-life counseling
Etonogestrel subdermal implants
Extended-cycle combined oral contraceptives
Heavy menstrual bleeding
Human papillomavirus vaccination
Lynch II syndrome
Obesity
Ovary preservation or removal at the time of hysterectomy
Patient with sexual partner who has genital herpes
Perioperative cardiac evaluation
Prevention of human papillomavirus transmission
Recommendations to prevent osteoporosis
Recurrent urinary tract infection
Risk of acquiring human immunodeficiency virus after sexual assault
Role of hormone therapy in menopause
Smoking cessation

ETHICAL AND LEGAL ISSUES


Discussion of medical errors
Ethical management of unexpected surgical findings
Patient privacy: electronic communication and office practice
Peer review of operative complications
Recognition of physician burnout

OFFICE PROCEDURES
Coding for office-based procedures
Patient privacy: electronic communication and office practice
Patient safety in the office
Recognition of physician burnout

A complete subject matter index appears at the end of the Critique Book.
Instructions

This book contains several types of questions. You will find special directions for each
question type inside the book. Be sure that you understand the directions before answering
any questions.
If you choose to participate in PROLOG for continuing medical education credit, your
answers should be recorded on the separate answer sheet enclosed. Directions for recording
your answers are included on the answer sheet. The registration code is printed on your
answer sheet and is already encoded into the computer-readable grid. You do not need to fill
out a registration code grid to identify your paper. The example below identifies PROLOG
user #E20024.

To ensure that your results are returned promptly, please complete your name and address
on the answer sheet. We suggest that you keep a personal record of both your registration
code and your answers so that you may check the individualized performance report that is
returned to you. If you are an ACOG member, please fill in your ACOG ID number. Extra
spaces can be left blank.
The completed answer sheet should be submitted for scoring in the postage-paid return
envelope enclosed with this package. Please allow 4–6 weeks for score sheets to be processed.
If you do not wish to submit your answers for scoring, you will not receive credit. The
Critique Book enclosed in this package will, however, enable you to score your own test.

xiii
DIRECTIONS: Each of the questions or incomplete statements below is followed by suggested
answers or completions. Select the ONE that is BEST in each case, and fill in the circle containing
the corresponding letter on the answer sheet.

1
A 52-year-old menopausal woman comes to your office for her annual examination. She has not
experienced spotting or vaginal bleeding since menopause 18 months ago. Her sister has a history
of cervical cancer. Her Pap test results are “satisfactory with no malignancy” with the notation
of “endometrial cells present in a woman older than 40 years.” The next step in her evaluation is

(A) endometrial assessment


(B) high-risk human papillomavirus testing
(C) repeat cervical cytology in 6 months
(D) colposcopy with endocervical curettage

2
A 16-year-old patient experienced acute back pain after a sports-related injury. The back pain
resolved with physical therapy; however, imaging studies obtained during her evaluation suggested
an adnexal mass. Ultrasonography is performed and reveals the image shown in Figure 2-1. The
most likely diagnosis is

(A) mature cystic teratoma


(B) endometrioma
(C) hydrosalpinx
(D) ectopic pregnancy
(E) hemorrhagic ovarian cyst

FIG. 2-1

Note: See Appendix A for a table of normal values for laboratory tests. 1
2 PROLOG

3
On postoperative day 4 after placement of a suburethral sling for urinary stress incontinence,
your patient has a postvoid residual volume of 250 mL. Before discharge from the hospital, the
postvoid residual volume was 300 mL, and the patient was discharged with an indwelling bladder
catheter. The surgery and postoperative course were otherwise uncomplicated. The best next step
in management is to

(A) release the sling


(B) start antibiotics
(C) replace the indwelling bladder catheter
(D) start intermittent self-catheterization
(E) perform urethral dilation

4
A 12-year-old girl is referred to your office by her pediatrician. She arrives with her mother. She
experienced her first menstrual cycle 14 months ago and has had irregular, heavy menstrual periods
since that time. She had no abnormal bleeding at the time of a tonsillectomy. She has no family
history suggestive of blood disorders. Her vital signs are within normal limits: quantitative human
chorionic gonadotropin level, less than 5 mIU/mL; hemoglobin level, 11.1 g/dL; hematocrit,
33.2%; mean corpuscular volume, 75 cubic micrometers; platelet count, 342 × 103 per microliter;
prothrombin time, 12.2 seconds; partial thromboplastin time, 27.4 seconds; and thyroid-stimulating
hormone level, 3.4 mIU/L. The best next step in management is

(A) iron supplementation


(B) office pelvic examination
(C) additional coagulation studies
(D) pelvic ultrasonography
(E) oral contraceptives

5
A 29-year-old obese nulliparous woman underwent an open myomectomy through a vertical mid-
line incision. Two weeks after the surgery, she comes to your office to have the incision inspected.
She has not experienced fever, but reports incisional pain, redness, and warmth. The bowel and
bladder functions are normal. Her vital signs are unremarkable and the incision is intact with mod-
erate periumbilical drainage, erythema, and tenderness. The best next step in her management is

(A) intravenous antibiotics


(B) oral antibiotics
(C) computerized tomography of the abdomen and pelvis
(D) explore the incision
Gynecology and Surgery 3

6
A 25-year-old woman is taken to the operating room for removal of an 8-cm complex adnexal mass
(Fig. 6-1). Inspection at the time of laparoscopy shows normal appearing diaphragms, peritoneal
surfaces, and omentum. The contralateral ovary has a 2-cm cyst and no excrescences. Pelvic wash-
ings and laparoscopic cystectomy are completed, and the cyst is removed without rupture. The
frozen-section diagnosis reveals a serous tumor of low malignant potential. The most appropriate
surgical procedure for this patient is

(A) no other procedure


(B) cystectomy of contralateral ovary
(C) peritoneal biopsies
(D) omentectomy and peritoneal biopsies
(E) omentectomy plus pelvic and paraaortic lymphadenectomy

FIG. 6-1

7
A 33-year-old woman, gravida 1, para 1, recently stopped breastfeeding. She comes to your office
to inquire about interval contraception. She had a normal spontaneous vaginal delivery and gave
birth to a healthy, term infant 12 months ago. The pregnancy was complicated by gestational diabe-
tes mellitus (GDM) and a seizure disorder. She is currently taking carbamazepine. Her past surgical
history includes a laparoscopic salpingostomy for ectopic pregnancy 5 years ago and malabsorptive
bariatric surgery. In the past, she has used only condoms and wishes to avoid irregular anovulatory
bleeding, but does not want to use an injectable contraceptive method. You counsel that her best
choice for contraception is

(A) levonorgestrel intrauterine device (IUD)


(B) cyclic combination oral contraceptive pills
(C) contraceptive rod
(D) extended-cycle oral contraceptive pills
(E) hysteroscopic bilateral tubal occlusion
4 PROLOG

8
A 65-year-old woman undergoes robotic-assisted laparoscopic abdominal–sacral colpopexy and
transobturator tension-free vaginal tape procedure for symptomatic pelvic organ prolapse and
urinary incontinence. The surgical time was 4.5 hours and estimated blood loss was 100 mL. On
postoperative day 1, her vital signs are stable and urine output is 40 mL/hr. The Foley catheter is
removed. On ambulating to the toilet, she experiences right foot drop, gait instability, and numb-
ness over her lateral leg and dorsum of the foot. The most appropriate next step in managing her
gait instability is

(A) physical therapy


(B) electromyography of the right leg
(C) magnetic resonance imaging of the pelvis
(D) duplex ultrasonography of the right leg

9
Your office staff expresses concern when a colleague becomes increasingly more irritated after
patient encounters. This behavior escalated after the practice administrator added ten additional
new patient slots per week and has opened the office an hour earlier to accommodate these addi-
tional appointments. The staff worries that the physician’s “bad attitude” might have a negative
effect on patient care, satisfaction, and safety. When you discuss the staff’s concerns with the
physician, the physician expresses indifference toward work and patient care. You suspect physi-
cian burnout. The factor most strongly linked to avoiding physician burnout is

(A) control over one’s schedule and hours worked


(B) earlier retirement age
(C) specialty choice
(D) level of financial compensation

10
A 67-year-old woman comes to your office with dyspareunia and vaginal dryness. She has been
sexually abstinent for the past 10 years because of her late husband’s chronic health concerns.
Previously, she enjoyed sexual activities, including vaginal penetration. She is recently widowed
and has a new partner. However, she finds she is unable to tolerate vaginal penetration because of
severe pain. Her history is notable for urinary incontinence, postherpetic neuropathy, and consti-
pation controlled by medication. Vulvar and vaginal examinations reveal smooth, pale epithelium
(Fig. 10-1; see color plate). Microscopic examination shows scant small, round, immature para-
basal cells. Vaginal pH is 5. You do not observe clue cells, candidiasis, or increased inflamed cells.
The most likely diagnosis is

(A) vaginal atrophy


(B) lichen sclerosus
(C) lichen planus
(D) desquamative inflammatory vaginitis
(E) pemphigus vulgaris
Gynecology and Surgery 5

11
A 19-year-old woman telephones your office to report that she had unprotected intercourse 4 days
ago. Her last menstrual period was 10 days ago and she is concerned that she may become preg-
nant. You counsel her that the most effective recommended emergency contraception for her is

(A) copper intrauterine device (IUD)


(B) oral levonorgestrel
(C) levonorgestrel IUD
(D) oral mifepristone
(E) oral ulipristal acetate

12
An 18-year-old woman comes to the office for follow-up of a right adnexal mass noted on com-
puted tomography of the abdomen and pelvis after a motor vehicle accident. She reports neither
abdominal nor pelvic pain. She has regular menstrual cycles and is using oral contraceptives. A
urine pregnancy test result is negative. Ultrasonography reveals a right adnexal mass of 3 cm in
largest diameter (Fig. 12-1) with ovarian volume of 25 cm3. She has a history of laparoscopic right
ovarian cystectomy for a mature cystic teratoma 3 years ago. The best next step in the management
of this mass is

(A) CA 125 testing


(B) laparoscopic resection
(C) observation
(D) magnetic resonance imaging (MRI)

FIG. 12-1
6 PROLOG

13
A 24-year-old nulliparous woman is referred to you with severe dysmenorrhea for the past 3 years.
She is incapacitated for 1–3 days at each menses. Trials of combined oral contraceptives, ibuprofen,
naproxen, and celecoxib have not relieved the pain. She has been married for 2 years and has been
trying to conceive for almost 1 year without success. The most appropriate management is

(A) laparoscopic uterine nerve ablation


(B) depot leuprolide
(C) noncyclic combined oral contraceptives
(D) diagnostic laparoscopy
(E) opioid analgesic

14
A 29-year-old woman, gravida 2, para 2, presents to discuss management after a cervical loop elec-
trosurgical excision procedure. The pathology report reveals adenocarcinoma in situ (AIS) with a
positive endocervical margin. She desires to maintain fertility. The best next step in management
for this patient is

(A) colposcopy with biopsy


(B) cytology and high-risk human papillomavirus (HPV) in 4–6 months
(C) endocervical curettage
(D) cold knife cone biopsy
(E) simple hysterectomy

15
A 38-year-old woman, gravida 3, para 3, comes to your office for follow-up of a loop electro-
surgical excision procedure (LEEP). She has no significant past medical history but smokes five
cigarettes per day. She has had seven lifetime sexual partners. Currently, her only sexual partner is
her husband of 5 years. She has undergone a bilateral tubal ligation. The result of her last human
immunodeficiency virus (HIV) test was negative 2 years ago during prenatal care. She also had
normal Pap test results until the most recent one, which was positive for a high-grade squamous
intraepithelial lesion (HSIL). Colposcopic biopsy confirms cervical intraepithelial neoplasia
(CIN) 3. Endocervical curettage result is negative. The LEEP test result is CIN 3 with positive
ectocervical margins. The best next step in this patient’s management is

(A) hysterectomy
(B) Pap test in 4–6 months
(C) cryotherapy
(D) treatment with topical imiquimod
Gynecology and Surgery 7

16
You perform a cystoscopy on a 65-year-old woman after you complete a total vaginal hysterec-
tomy and uterosacral ligament suspension for vaginal prolapse. The patient received intravenous
(IV) indigo carmine, and now you observe an immediate efflux of dye from the right ureter. After
30 minutes, no efflux is observed from the left ureter. The next step in management should be to

(A) continue observation for another 15 minutes


(B) administer IV furosemide
(C) place a ureteral stent on the left side
(D) remove the most lateral and posterior suture on the left

17
A 40-year-old woman, gravida 2, para 2, returns to your office 3 years after you performed a total
abdominal hysterectomy and bilateral salpingo-oophorectomy for stage IV endometriosis with
severe adhesive disease. She has been experiencing right-sided abdominopelvic pain that is severe
and similar to the pain she had before the hysterectomy. Her only medication is an estradiol patch.
On examination, she has tenderness in the right lower quadrant and at the right vaginal apex.
Ultrasonography suggests a 2 cm × 1.5 cm cyst at the right vaginal apex. The patient requests sur-
gery. The most appropriate next step is

(A) laparoscopic removal of the pelvic cyst


(B) depot leuprolide treatment for 6 months
(C) colonoscopy
(D) stop estradiol therapy and obtain follicle-stimulating hormone (FSH) level
(E) referral to a chronic pain clinic

18
A woman who is known to have human immunodeficiency virus (HIV) is undergoing an emer-
gency cesarean delivery. Her viral titer is greater than 1,500 copies per milliliter. Postexposure
prophylaxis should be offered for

(A) a penetrating finger injury with a visibly bloody scalpel


(B) a blood splash onto the surgeon’s unprotected forehead
(C) amniotic fluid saturating through the surgeon’s sleeve
(D) a nonpenetrating needlestick through glove during myometrial closure
(E) unprotected skin exposure to sputum during extubation
8 PROLOG

19
A 23-year-old woman comes to the office and requests removal of her radioopaque single-rod
implantable contraceptive 2 months after insertion because she wishes to conceive with a new
partner. Palpation at the site of insertion is unsuccessful at localizing the rod. The patient reports
neither infection nor trauma at the insertion site. Her pregnancy test result is negative. The best next
step in the care of this patient is

(A) serum etonogestrel level measurement


(B) follow-up in 6 months if not pregnant
(C) incision and exploration
(D) X-ray of the arm

20
A 69-year-old woman comes to your office and reports increasing urinary incontinence over the
past year. The episodes have occurred with coughing or when she has been unable to make it to
the bathroom in time. She has been menopausal for 12 years and has no chronic medical problems.
The answers on The 3 Incontinence Questions (3 IQ) assessment tool suggests she may have mixed
incontinence. In addition to the screening questionnaire, a complete history, physical examination,
urinalysis, and urine culture, the next step in this patient’s management should be

(A) a voiding diary


(B) anticholinergic therapy
(C) urodynamic testing
(D) Kegel exercises
(E) a pessary

21
You were recently notified about a possible legal action that might be filed against you regarding
a patient who had a serious complication after surgery. You have kept numerous e-mail messages
with this patient during her difficult postoperative recovery. Her attorney has sent a letter, which
includes her signed consent, for release of her medical records. Regarding the e-mail communica-
tion between you and the patient, the proper course of action is to

(A) delete them from your computer


(B) supply them only if specifically requested
(C) supply them only to your attorney
(D) send them with the medical records
Gynecology and Surgery 9

22
A 34-year-old woman comes to your office for her annual health maintenance examination. She
has been smoking one pack of cigarettes per day for the past 7 years. During the interview, she
mentions that she has been thinking about the effect of smoking on her health and is considering
quitting. She has tried to quit in the past. The patient’s current stage of readiness to change her
smoking behavior is

(A) precontemplation
(B) contemplation
(C) action
(D) maintenance
(E) relapse

23
Your hospital administrator asks you to give a presentation to the medical staff on the topic of medi-
cal liability. During the question-and-answer period, audience members ask if full disclosure will
decrease the number of professional liability suits, reduce average settlement amounts, improve
patient ratings of quality of care, or produce negative publicity about the care provided by the hos-
pital. You reply that, of those questions, full disclosure has been shown to

(A) decrease the number of professional liability suits


(B) have no effect on the average settlement amount
(C) decrease the patient’s rating of quality of care
(D) increase the number of negative media reports

24
A 44-year-old woman, gravida 2, para 2, comes to your office for a well-woman examination.
Her body mass index (BMI) is 35 (calculated as weight in kilograms divided by height in meters
squared). You note that the patient has gained 10 kg (22 lb) over the past 2 years. She has no
illnesses, but her family history is significant for diabetes mellitus, coronary artery disease, and
hypertension. The first intervention to facilitate weight loss in this patient should be

(A) counsel aboout the importance of weight loss


(B) refer for bariatric surgery
(C) prescribe weight loss medication
(D) screen for metabolic risk
10 PROLOG

25
A 48-year-old woman comes to your office after a visit to the emergency department for severe left
flank pain during which ultrasonography showed a 3.96-cm left ovarian mass (Fig. 25-1). She has
not experienced any change in appetite, early satiety, or increased abdominal girth. The ultrasono-
graphic marker that is most concerning for an ovarian neoplasm in this patient is

(A) cyst size


(B) fluid echogenicity
(C) cyst wall thickness
(D) solid excrescence

Left Ovary Transverse

FIG. 25-1

26
A 62-year-old woman, gravida 2, para 2, is referred to you for recurrent postmenopausal bleed-
ing. She reports a history of light bleeding 6 months ago. At that time, her primary care physician
ordered pelvic ultrasonography that demonstrated a 4-mm endometrial thickness. The patient tells
you that she had no further bleeding until 3 weeks ago. She reports that she has no other medical
problems and that she is not taking any prescription medications. Her last Pap test was approxi-
mately 1 year ago and the result was normal. She has no history of cervical dysplasia. Her body
mass index is 36 (calculated as weight in kilograms divided by height in meters squared) and the
pelvic examination is notable only for vaginal atrophy. You counsel her that the best next step is
to perform

(A) pelvic examination in 6 months


(B) hysteroscopy with dilation and curettage
(C) saline sonohysterography
(D) an endometrial biopsy
Gynecology and Surgery 11

27
A 30-year-old multiparous woman with a BRCA 2 mutation comes to your office for her annual
gynecologic examination. She wishes to decrease her risk of developing ovarian cancer over the
next 10 years. She does not desire pregnancy now or in the future. You advise her that the most
appropriate contraceptive management is

(A) bilateral salpingo-oophorectomy


(B) levonorgestrel intrauterine device
(C) combined oral contraceptives
(D) bilateral tubal ligation

28
An 18-year-old woman comes to the emergency department with an 8-hour history of severe right
lower quadrant pain. She has a history of a prior left salpingo-oophorectomy performed for ovarian
torsion 3 years ago. Ultrasonography shows a 3-cm simple ovarian cyst and concern for adnexal
torsion. Intraoperatively, the remaining ovary appears ischemic with a blue-black color. The most
appropriate surgical intervention is

(A) proceed with removal of the right fallopian tube and ovary
(B) perform ovarian cystectomy
(C) detorse the ovary and perform an oophoropexy
(D) perform hysterectomy and right salpingo-oophorectomy

29
A 38-year-old nulligravid woman comes to your office with lower abdominal cramping pain for
the past 2 years. The pain is noted to be worse during her menses, which are regular in frequency,
duration, and amount. Her last menstrual period was 3 weeks ago. She reports bloating and rectal
pressure that is partially relieved by defecation, occasional diarrhea, and passage of mucus for
2 years. She has experienced occasional pain with urination for 2 months. Urinalysis results are
negative for white blood cells and leukocyte esterase. Pelvic examination reveals a 5-cm left pelvic
mass. The next diagnostic test for this patient should be

(A) laparoscopy
(B) computed tomography (CT)
(C) colonoscopy
(D) ultrasonography
(E) magnetic resonance imaging (MRI)
12 PROLOG

30
A 34-year-old woman comes to your office after feeling a lump in her left breast for the past
4 weeks. A breast examination reveals a mobile, cystic 2-cm mass. The patient agrees to a fine-
needle aspiration that yields straw-colored fluid. Four months later, the patient returns and reports
that the mass has reoccurred. Your examination confirms a 3-cm soft, mobile cystic lesion in the
area of the prior aspiration site. The best next step in evaluation of this mass is

(A) reassurance with repeat examination in 6 months


(B) ultrasonography
(C) repeat aspiration
(D) mammography
(E) dietary modification

31
A 77-year-old woman, gravida 3, para 3, comes to your office with a vaginal bulge and urinary
obstruction that requires a Foley catheter for bladder drainage. She has a history of severe coronary
artery disease and chronic obstructive pulmonary disease. She underwent a total vaginal hyster-
ectomy with anterior repair for prolapse at age 40 years. Recently, she had an unsuccessful pes-
sary trial because of expulsion of the pessary with defecation. She was continent of urine without
urinary obstruction or leakage when the pessary was in place. She has been widowed for 10 years
and is not sexually active. A preoperative evaluation confirms that she is a poor surgical candidate
with significant risk for surgical anesthesia. On examination, she has a large central defect. The
best surgical option is

(A) paravaginal repair


(B) traditional midline colporrhaphy
(C) uterosacral ligament suspension
(D) sacrospinous ligament fixation
(E) Le Fort partial colpocleisis

32
A 42-year-old woman underwent a total vaginal hysterectomy 10 days ago. Yesterday she began to
experience vaginal fluid leakage that requires the use of pads. She has no vaginal bleeding, fever,
chills, dysuria, or urinary frequency. The most appropriate next step is

(A) urine culture


(B) urodynamic testing
(C) office instillation of dye into bladder
(D) check a postvoid residual
Gynecology and Surgery 13

33
A 17-year-old sexually active female comes to your office with her mother for contraceptive coun-
seling. She has been using depot medroxyprogesterone acetate (DMPA) for birth control over the
past 18 months. She wishes to continue the DMPA. She has no history of bone fractures and is in
good health. Currently, she does not take any prescription medications, but in the past she had a dif-
ficult time remembering to take oral medications. Her mother has read about the thinning of bones
associated with DMPA and inquires about bone mineral density (BMD) testing. After discussion
of her risk factors for bone loss, the best next step in her management is

(A) dual energy X-ray absorptiometry (DXA)


(B) treat with bisphosphonates
(C) continue with DMPA
(D) switch to oral contraceptives
(E) check bone-turnover metabolites

34
A 33-year-old nulligravid woman comes to your office with Pap test results reported as atypical
glandular cells, not otherwise specified (AGC–NOS). She has a history of irregular menses every
7–10 weeks. She has no medical problems and is not currently in a sexual relationship. Her last Pap
test 3 years ago yielded a normal result; human papillomavirus (HPV) testing also yielded a nor-
mal result. Her family history is significant for colon cancer (it was diagnosed in her father at age
51 years). On physical examination, her height is 1.6 m (64 in), weight 106.6 kg (235 lb), blood
pressure 125 mm Hg systolic, 75 mm Hg diastolic, and pulse 84 beats per minute. Pelvic examina-
tion yields a normal result. The best next step in management is

(A) repeat Pap test with reflex HPV testing in 6 months


(B) repeat HPV testing in 12 months
(C) loop electrosurgical excision procedure (LEEP)
(D) colposcopy, endocervical curettage, and endometrial biopsy

35
A 60-year-old woman undergoes an uncomplicated bilateral salpingo-oophorectomy for a 20-cm
benign mucinous cystadenoma. On postoperative day 1, you are contacted by the nursing staff and
informed that the patient has developed a fever of 38.4°C (101°F). The patient states that despite
the fever she feels fine. Her oxygen saturation is 92% on room air that improves to 98% with ambu-
lation. Physical examination findings are remarkable for decreased breath and bowel sounds, a
clean and intact incision and peripheral intravenous site, and trace bilateral pedal edema with bilat-
eral sequential devices and compression stockings in place. The best next step in management is

(A) aggressive pulmonary therapy


(B) blood and urine cultures
(C) broad-spectrum antibiotics
(D) therapeutic anticoagulation
(E) computed tomography of the chest
14 PROLOG

36
A 33-year-old woman, gravida 2, para 2, comes to your clinic with recurrent malodorous vaginal
discharge and vulvar irritation with symptoms every 1–2 months in the past year. She received a
diagnosis and was treated for bacterial vaginosis 4 times in the past 5 months. Each time, she had a
complete resolution of symptoms followed by recurrence after sexual intercourse. After treatment
of her recent infection, the best management plan is

(A) treat sexual partner(s)


(B) recommend weekly douching
(C) postcoital oral clindamycin
(D) metronidazole vaginal gel twice weekly

37
A 38-year-old woman, gravida 3, para 3, has a 4-month history of heavy menstrual bleeding. Past
surgical history is significant for three cesarean deliveries with the third procedure complicated by
a bowel injury. Laboratory evaluation includes a hemoglobin level of 9.2 g/dL. Sonohysterography
shows a 3.5-cm pedunculated fundal submucosal myoma with no intramural extension and a
uterine sagittal length of 10 cm. Endometrial biopsy reveals disordered, proliferative phase endo-
metrium. The patient desires future fertility. The best approach for management of this patient is

(A) hysteroscopic myomectomy


(B) nonresectoscopic endometrial ablation
(C) magnetic resonance imaging (MRI)-guided focused ultrasound ablation
(D) uterine artery embolization
(E) abdominal myomectomy

38
A 63-year-old woman is scheduled to undergo laparoscopic hysterectomy with bilateral salpingo-
oophorectomy and pelvic washings along with possible pelvic and paraaortic lymphadenectomy for
poorly differentiated endometrial adenocarcinoma. She has a history of morbid obesity with a body
mass index (BMI) of 42 (calculated as weight in kilograms divided by height in meters squared)
and poorly controlled diabetes mellitus. She has recently developed dyspnea and chest pressure
with walking. Her blood pressure is 160 mm Hg systolic, 100 mm Hg diastolic, and her heart rate
is 83 beats per minute. A laboratory evaluation reveals a fasting blood sugar level of 120 mg/dL,
hemoglobin A1c level of 7.5%, hemoglobin level of 10.5 g/dL, and serum creatinine level of 1.2
mg/dL. The chest X-ray result is normal. She had a normal result on screening colonoscopy 3 years
ago. You advise her that it is best to postpone the procedure because of the

(A) obesity
(B) hemoglobin A1c level
(C) blood pressure level
(D) dyspnea and chest pressure
(E) hemoglobin level
Gynecology and Surgery 15

39
A 35-year-old woman, gravida 2, para 2, comes to your office for her annual examination.
Recently, she received the diagnosis of Lynch II syndrome. Her mother had uterine cancer at age
52 years, her maternal uncle had colon cancer at age 47 years, and her older sister had ovarian
cancer at age 46 years. The best strategy to decrease the patient’s risk of gastrointestinal and gyne-
cologic cancer over the next 5 years is

(A) annual computed tomography examination of abdomen and pelvis


(B) annual colonoscopy and transvaginal ultrasonography with endometrial biopsy
(C) prophylactic colectomy with annual transvaginal ultrasonography
(D) prophylactic hysterectomy and bilateral salpingo-oophorectomy with annual
colonoscopy

40
A 38-year-old woman, gravida 4, para 4, comes to your office and requests contraception. She has
a 4-month history of irregular bleeding that lasts up to 17 days per month. Her past medical history
is significant for poorly controlled hypertension. On physical examination, her blood pressure (BP)
is 160 mm Hg systolic, 110 mm Hg diastolic and her body mass index (BMI) is 32 (calculated as
weight in kilograms divided by height in meters squared). A urine pregnancy test result is nega-
tive. Other laboratory values are normal except for a hemoglobin level of 11.3 g/dL. Endometrial
biopsy reveals a disordered, proliferative phase endometrium. The best contraceptive choice for
this patient is

(A) depot medroxyprogesterone acetate


(B) contraceptive vaginal ring
(C) combined oral contraceptives
(D) levonorgestrel IUD
(E) copper IUD

41
A 48-year-old woman undergoes a total laparoscopic hysterectomy and bilateral salpingo-oopho-
rectomy for abnormal uterine bleeding. The uterus is noted to be homogeneously enlarged at
10-week-of-gestation size with grossly normal appearing ovaries. The procedure is uncomplicated
with the exception of some bleeding at the vaginal cuff that is cauterized. On postoperative day 2,
the patient has right flank pain and intravenous pyelography reveals a dilated and obstructed distal
right ureter. The most likely surgical step that led to the ureteral injury in this patient is

(A) ligation of the infundibulopelvic ligament


(B) opening the broad ligament
(C) coagulation of the uterine vessels
(D) transection of the uterosacral ligament
(E) incision for colpotomy
16 PROLOG

42
A healthy 33-year-old woman is having a laparoscopic ovarian cystectomy for a symptomatic der-
moid cyst. A pneumoperitoneum is created under pressure less than 8 mm Hg and is increased for
trocar insertion. During insertion of the umbilical trocar, the anesthesiologist informs you that the
patient’s blood pressure has decreased suddenly to 70 mm Hg systolic, 30 mm Hg diastolic. The
patient’s pulse is 32 beats per minute and oxygen saturation is 90%. The most appropriate next step
in management is to

(A) continue laparoscopic procedure


(B) perform immediate laparotomy
(C) terminate procedure
(D) desufflate the abdomen

43
A 20-year-old woman visits your office for infertility. She and her partner have been unsuccessful
in attempting conception for 13 months. She has a history of Class III obesity with a body mass
index of 36 (calculated as weight in kilograms divided by height in meters squared) and polycystic
ovary syndrome (PCOS). She previously used oral contraceptives for birth control. After discon-
tinuing the oral contraceptives, she had four menstrual periods over the past 14 months. A 2-hour
plasma glucose level after ingestion of a 75-g glucose load was 160 mg/dL. The best first step in
the management of her infertility is

(A) metformin hydrochloride


(B) rosiglitazone
(C) weight loss
(D) clomiphene citrate
(E) exogenous gonadotropins

44
A 24-year-old woman presents to the emergency department with vaginal bleeding and pelvic pain
for 2 days. Her menses are irregular and she thinks her last menstrual period was 12 weeks ago.
She was treated for chlamydial infection 8 months ago. The examination is unremarkable except
for tenderness in the right lower abdominal quadrant. Laboratory values are as follows: blood
type, A+; hemoglobin level, 12.4 g/dL; hematocrit, 35%; white blood cell count, 13,000 cells per
microliter; platelet count, 86 × 103 per microliter; quantitative serum b-hCG level, 3,600 mIU/mL;
aspartate aminotransferase level, 60 units/L; alanine aminotransferase level, 52 units/L; and serum
creatinine level, 1.4 mg/dL. Pelvic ultrasonography reveals a 2.8-cm cystic mass with a yolk sac
within the right adnexa and no intrauterine pregnancy. No free fluid is observed. The most appro-
priate management is

(A) laparoscopic salpingostomy


(B) methotrexate
(C) repeat b-hCG level measurement in 48 hours
(D) laparotomy with salpingostomy
Gynecology and Surgery 17

45
A 43-year-old woman, gravida 2, para 2, comes to your office for follow-up of depression. Two
months ago, you prescribed a selective serotonin reuptake inhibitor (SSRI) for symptoms of depres-
sion, and her mood has now improved. She notes a decrease in orgasmic response; however, she
feels comfortable with the decreased response because her husband is currently deployed with the
military. She asks how long she will need to take the SSRI. The minimum suggested duration to
take the SSRI is

(A) 3 months
(B) 6 months
(C) 15 months
(D) 21 months
(E) indefinitely

46
A 62-year-old woman comes to your clinic with a 1-year history of severe dyspareunia and inability
to tolerate vaginal penetration for the past several months. Your examination reveals the vulvar and
oral findings shown in Fig. 46-1 (see color plate) and Fig. 46-2 (see color plate), respectively. Test
results are negative for Candida species. The most likely diagnosis is

(A) erosive lichen planus


(B) lichen sclerosus
(C) atrophic vaginitis
(D) desquamative inflammatory vaginitis
(E) Behçet disease
18 PROLOG

47
A 32-year-old woman comes to the emergency department with abdominal distension, abdominal
pain, and bilious vomiting at day 4 from after a laparoscopic ovarian cystectomy for a 14-cm
mature teratoma. Her temperature is 38.7°C (101.7°F). Her heart rate is 115 beats per minute.
Her blood pressure is 98 mm Hg systolic, 54 mm Hg diastolic. On examination, the abdomen is
distended and soft but diffusely tender with maximal tenderness over the umbilical port site. The
umbilical port site appears erythematous. Bowel sounds are hypoactive. Her total white cell count
is 22,000 cells per microliter with 12% neutrophils. Based on computed tomography (CT) of the
abdomen (Fig. 47-1), the best management course is

(A) surgery
(B) bowel rest with nasogastric suction
(C) small-bowel series
(D) ultrasonography
(E) intravenous antibiotic administration and observation

FIG. 47-1

48
A 50-year-old woman comes to your office to discuss hormone therapy (HT). Two months ago,
she underwent hysterectomy with bilateral salpingo-oophorectomy and appendectomy for bilateral
adnexal masses with a pathologic diagnosis of mucinous cystadenomas. She has no menopausal
symptoms but has heard about the “critical timing hypothesis” and is interested in beginning HT.
You explain to her that the reasons for initiating systemic HT should be limited to

(A) prevention of coronary heart disease


(B) treatment of osteoporosis
(C) treatment of menopausal symptoms
(D) prevention of dementia
(E) treatment of urinary incontinence
Gynecology and Surgery 19

49
You evaluate a 73-year-old woman, gravida 2, para 2, for low urine output that occurs 90 minutes
after total vaginal hysterectomy. The surgery was uneventful, with an estimated blood loss of
300 mL. She received 2,000 mL of isotonic crystalloid fluid during and after surgery. Before
surgery, she followed nil per os (NPO) instructions. Bladder volume was 70 mL when the Foley
catheter was placed at the conclusion of the surgery. Her past medical history is significant for
hypertension, well controlled with lisinopril, and diet-controlled type 2 diabetes mellitus. On exam-
ination, the patient has a temperature of 36.8°C (98.2°F), pulse of 100 beats per minute, respiratory
rate of 18 breaths per minute, and blood pressure of 136 mm Hg systolic, 82 mm Hg diastolic. Her
abdomen is soft and tender to palpation. No drainage or discharge from the vagina is present. The
Foley catheter is in place with 7 mL of yellow urine in the bag. You irrigate the catheter with return
of an equal amount of fluid. The next step in management is

(A) diagnostic laparoscopy


(B) cystoscopy
(C) computed tomography of abdomen and pelvis
(D) intravenous furosemide
(E) fractional excretion of sodium (FENa)

50
A 27-year-old primiparous woman consults you regarding management of the abnormal cervical
cytology result. She had normal Pap test results until 12 months ago when a Pap test showed that
she had a low-grade squamous intraepithelial lesion (LSIL). A colposcopic biopsy showed cervical
intraepithelial neoplasia (CIN) 1. Twelve months later, she received a positive human papillomavi-
rus (HPV) test result. The next step in the management of this patient is

(A) cryotherapy
(B) loop electrosurgical excision procedure
(C) testing for HPV in 1 year
(D) Pap test in 6 months
(E) colposcopy

51
A 41-year-old woman with abnormal uterine bleeding comes to you for a hysterectomy. Before the
surgery, you review the consent form with her. You advise her of the risks and benefits of the pro-
cedure, discuss the alternatives to the surgical procedure, and answer her questions. The procedure
listed on the surgical consent is “Laparoscopic Total Hysterectomy.” During the surgery, a trocar
injures the internal iliac artery. You request a scalpel and ask the circulator to set up for an open
procedure. The circulator states that the patient has not consented to laparotomy. In reference to the
concerns regarding the consent form, you inform the circulator that

(A) the medical emergency justifies the change


(B) the patient’s family in the waiting room should be asked for consent
(C) the risk management personnel should be contacted for guidance
(D) another gynecologic surgeon agreeable to the change should be called
20 PROLOG

52
A 17-year-old patient comes to your office for the annual school physical examination. On review-
ing her immunization record, you realize that she received two doses of the bivalent human pap-
illomavirus (HPV) vaccine last year but missed her appointment for the third dose. The patient
informs you that she has been sexually active with two prior partners. Regarding HPV vaccination,
you counsel her that the most appropriate next step is to

(A) restart the vaccination series


(B) do nothing because she has likely been exposed to HPV
(C) administer one more dose of the bivalent vaccine
(D) administer the quadrivalent vaccine
(E) perform HPV testing before further vaccination

53
A 32-year-old woman comes to your office with recurrent yeast infections that began 14 months
ago and increased in frequency in the past 4 months. The patient notes that the use of over-the-
counter antifungals initially provided benefit. However, recently, their use has worsened the itching
and discomfort. Her main concern is intractable itching that wakes her up at night. Her husband
has not been affected. Examination of the vulva reveals the findings shown in Fig. 53-1 (see color
plate). Microscopy and cultures yield negative results for bacterial vaginosis and Candida. The
most likely diagnosis is

(A) lichen simplex chronicus


(B) lichen sclerosus
(C) vulvar intraepithelial neoplasia (VIN)
(D) inverse psoriasis
(E) Paget disease

54
A 24-year-old woman comes to the emergency department with vaginal bleeding. She had a suction
curettage for a complete molar pregnancy 11 weeks ago. The preevacuation serum b-hCG level was
285,000 mIU/mL. On physical examination, she is hemodynamically stable, and a small amount
of clotting blood is seen in the vagina. Her quantitative b-hCG level last week was 2,485 mIU/mL
and is found to be 2,450 mIU/mL today. The most appropriate management is

(A) methotrexate therapy


(B) combination chemotherapy
(C) quantitative b-hCG measurement in 1 week
(D) suction curettage
(E) transvaginal pelvic ultrasonography
Gynecology and Surgery 21

55
A 42-year-old woman comes to your office with heavy menstrual bleeding for the past 9 months
despite treatment with combination oral contraceptives. Recent ultrasonography revealed an intra-
mural anterior fundal myoma that measured 7.5 cm × 8.1 cm. She recently read about uterine artery
embolization as a treatment option. You counsel her that the most common complication associ-
ated with uterine artery embolization is

(A) readmission
(B) postembolization syndrome
(C) sepsis
(D) unsuccessful bilateral embolization
(E) prolonged hospital stay

56
A 28-year-old woman comes to the emergency department at your hospital 12 hours after being
sexually assaulted. She reports vaginal and anal penetration, and physical examination identifies
numerous lacerations. The sexual assault involved one unknown perpetrator. The patient’s immu-
nization record is unavailable. When counseling the patient about emergency contraception and
prophylaxis against sexually transmitted infections (STIs), you advise her against routine prophy-
laxis for

(A) pregnancy
(B) emotional trauma
(C) human immunodeficiency virus (HIV)
(D) hepatitis B virus

57
You participate in a departmental Quality Assurance and Improvement Committee meeting at your
hospital. One of the cases involves a ureteral injury during a surgical procedure carried out by a
urogynecologist. The division director of urogynecology, who is not a member of the committee,
sees you in the cafeteria and asks you about the discussion of this case. You inform her that you

(A) will forward her the minutes from the meeting


(B) cannot discuss the meeting with her
(C) can tell her about the discussion in a private place
(D) can relate the outcome of the meeting but no other details
22 PROLOG

58
A 48-year-old perimenopausal woman, gravida 2, para 2, comes to your clinic for follow-up of
decreased libido and hypoactive sexual disorder. She has no medical problems and takes no medi-
cations; her examination yields normal results. She notes no improvement after four sessions of
couple’s counseling and reports a good relationship with her husband. She would like to initiate
medical therapy to see if she can improve her sex drive while she continues counseling. The best
recommendation to improve her libido is

(A) phosphodiesterase inhibitor


(B) selective serotonin reuptake inhibitor
(C) psychotropic agent
(D) estrogen therapy
(E) testosterone therapy

59
A 45-year-old woman comes to your office for her annual gynecologic examination. She reports
that her mother died of ovarian cancer at age 55 years. No other breast, ovarian, endometrial, or
colon cancer occurred in her family. When discussing with her the pros and cons of screening for
ovarian cancer, you advise her that the screening test that might have the most clinical benefit for
her is

(A) annual CA 125 testing


(B) ovarian cancer blood test
(C) transvaginal ultrasonography
(D) inhibin test
(E) pelvic magnetic resonance imaging

60
A 23-year-old woman, gravida 4, para 4, comes to your office at 6 weeks postpartum for hystero-
scopic sterilization with a metallic microinsert. During the review of her medical records, you
notice that her history is complicated by sensitivity to nickel and that she had a finding of atypical
squamous cells of undetermined significance (ASC-US) on a recent Pap test. She also reports that
when she underwent intravenous pyelography after a complicated cesarean delivery, she developed
chest tightness and a rash. The contraindication to metallic microinsert hysteroscopic sterilization
in this patient is

(A) her age


(B) allergy to contrast dye
(C) nickel sensitivity
(D) finding of ASC-US on Pap test
(E) number of weeks postpartum
Gynecology and Surgery 23

61
A 67-year-old woman comes to your office for evaluation of urinary incontinence. She reports no
symptoms associated with cough or Valsalva maneuvers, but has repeated episodes of urgency
before leakage. Her physical and pelvic examination results are within normal limits. Review of
symptoms reveals recent blurring of vision and eye pain, which the patient attributes to stress.
You obtain a urine culture, which reveals no growth. Before initiation of a trial of anticholinergic
therapy, the most appropriate next step is

(A) ophthalmologic evaluation


(B) hemoglobin A1c test
(C) Kegel exercises
(D) urodynamics

62
You are consulted about an 11-year-old patient who was admitted to the hospital for an inability
to tolerate solids or liquids, a 3-day history of acute and severe menstrual bleeding at menarche,
and newly diagnosed immune thrombocytopenic purpura (ITP). The girl has never been sexually
active nor has she had a pelvic examination. She is currently undergoing a transfusion of packed
red blood cells and platelets and is receiving intravenous (IV) fluids, immunoglobulin, and corti-
costeroids. She is hemodynamically stable with splenomegaly. The physical examination revealed
normal external genitalia with a virginal introitus and approximately 500 mL of bright red blood on
a pad. Transabdominal pelvic ultrasonography yields normal results. A urine pregnancy test result
is negative. Hemoglobin level is 6 g/dL and platelet count is 12 × 103 per microliter. The best next
step in management is

(A) depot medroxyprogesterone acetate


(B) IV conjugated equine estrogens
(C) combined oral contraceptives
(D) oral progestin
(E) levonorgestrel IUD

63
A 50-year-old woman is scheduled for a total laparoscopic hysterectomy and bilateral salpingo-
oophorectomy for simple endometrial hyperplasia. Her medical history is significant for a 20-pack-
year smoking history and a body mass index of 40 (calculated as weight in kilograms divided
by height in meters squared). Her obstetric history is remarkable for two vaginal births and one
cesarean delivery for a breech presentation. The best option to minimize her risk for a perioperative
thromboembolic event is

(A) smoking cessation


(B) prophylactic warfarin sodium
(C) unfractionated heparin
(D) change from surgery to laparotomy
24 PROLOG

64
A 45-year-old woman has been referred to you for a complex left adnexal mass, left lower quadrant
and flank pain, hematuria, rectal bleeding, and anemia. Two years ago, she had a total laparoscopic
hysterectomy with bilateral salpingo-oophorectomy for stage IV endometriosis. She is otherwise
healthy. A left pararectal mass is palpated on examination. Computed tomography (CT)–intrave-
nous pyelography reveals extrinsic compression of the left ureter with severe left hydronephrosis
and a 6-cm complex left pelvic mass. Laboratory evaluation shows a hemoglobin level of 10.7 g/dL.
Her urologist places a retrograde stent into the left ureter. Cystoscopy yields normal results.
The remainder of her medical history is unremarkable. The next step in the management of this
patient is

(A) surgical excision of the pelvic mass


(B) retrograde pyelography
(C) nephrostomy
(D) colonoscopy

65
A 34-year-old woman, gravida 1, para 1, comes to your office with irregular bleeding that began
3 months ago. Physical and pelvic examinations yield normal results. The urine pregnancy test
result is negative. Sonohysterography shows a 1-cm intrauterine cavitary mass with a feeding
vessel through its stalk. You recommend office hysteroscopy with polypectomy preceded by a
prostaglandin cervical ripening agent administration. During insertion of the hysteroscope, the
patient becomes hypotensive and bradycardic. She maintains her oxygen saturation. She reports
no chest pain or shortness of breath, just cramping. Figure 65-1 (see color plate) shows the image
you see on the video monitor. Immediate fluid deficit of 500 mL of normal saline solution occurs.
Blood pressure and heart rate normalize with removal of the hysteroscope. The best next step in
management is

(A) completion of the procedure


(B) ventilation–perfusion lung imaging
(C) observation
(D) chest computed tomography angiography
(E) diagnostic laparoscopy
Gynecology and Surgery 25

66
A 55-year-old woman comes to the emergency department 2 weeks after an uncomplicated laparo-
scopic hysterectomy and bilateral salpingo-oophorectomy. She has been experiencing fever, chills,
and pelvic pain but reports normal bowel and bladder functions. Examination is remarkable for an
intact vaginal cuff with a 5-cm fluctuant mass noted with mild diffuse abdominal pain. A tempera-
ture of 38.4°C (101°F) is noted with otherwise stable vital signs. Laboratory results are remarkable
for an elevated white blood cell count. Computerized tomography demonstrates a 10 cm × 10 cm
× 10 cm pelvic fluid collection consistent with a postoperative abscess superior to the vaginal cuff.
The most important next step in management is

(A) oral antibiotics


(B) hospital observation
(C) intravenous antibiotics
(D) drainage of the abscess

67
A 47-year-old woman, gravida 2, para 2, is scheduled for laparoscopy-assisted vaginal hyster-
ectomy for abnormal uterine bleeding. During her recent office visit, she told you she had an
allergic reaction to penicillin with a mild body rash that resolved with over-the-counter diphen-
hydramine. The best step in management to reduce perioperative infection risk is to administer

(A) clindamycin
(B) metronidazole and a fluoroquinolone
(C) vancomycin
(D) cefazolin

68
A 60-year-old woman, gravida 3, para 3, with a history of abnormal uterine bleeding recently
underwent an abdominal hysterectomy and bilateral salpingo-oophorectomy for uterine myomas.
Her past medical history is significant for hypertension and recurrent bacterial vaginosis. Her
obstetric and gynecologic history is remarkable for three normal spontaneous vaginal births and
a lifetime history of normal Pap test results. She inquires about future Pap tests during her annual
examinations. In regard to future Pap tests, you advise her that she should

(A) have annual tests


(B) have tests every 3 years
(C) discontinue tests
(D) include testing for human papillomavirus (HPV) every 5 years
26 PROLOG

69
You have just performed a hysterectomy with bilateral salpingo-oophorectomy (BSO) in a 33-year-
old woman for treatment of severe pelvic pain associated with stage IV endometriosis because she
desired definitive surgery. At the time of surgery, you were able to remove all of the visible endo-
metriosis. In regard to hormone therapy (HT) after the procedure, the most appropriate choice is

(A) prescribe combined estrogen–progestin postoperatively


(B) delay estrogen administration for at least 12 months
(C) administer depot leuprolide injections postoperatively
(D) prescribe estrogen postoperatively

70
An 85-year-old woman, gravida 4, para 4, comes to your office for treatment of a bothersome vagi-
nal bulge and accompanying urinary retention. Her medical history is significant for hypertension,
coronary artery disease, and emphysema, for which she requires supplemental oxygen at home. Her
prior surgical procedures include vaginal hysterectomy at age 45 years and coronary artery bypass
grafting 5 years ago. On pelvic examination, she has significant anterior wall and apical prolapse,
with minimal posterior wall prolapse. Results of a pelvic organ prolapse quantification examination
are consistent with Stage III pelvic organ prolapse. When the anterior prolapse is reduced in the
office, she has no incontinence. She desires to retain coital ability. The most appropriate next step
in managing this patient is

(A) observation
(B) support pessary
(C) suburethral sling procedure
(D) anterior repair
(E) colposacropexy

71
A 33-year-old woman, gravida 3, para 3, comes to your office for her annual gynecologic exami-
nation. She is married and reports being in good health. The patient has normal breast and pelvic
examination results. You perform a Pap test and send the specimen for cytology and human papil-
lomavirus (HPV) co-testing. The Pap test result is normal, but the HPV test result is positive. You
counsel the patient that the most appropriate next step is to

(A) repeat the Pap test with HPV cotesting in 3 years


(B) perform colposcopy
(C) repeat the Pap test in 1 year
(D) perform HPV testing in 1 year
(E) test for HPV 16 and 18 genotypes
Gynecology and Surgery 27

72
A 43-year-old multiparous woman comes to your office for her annual gynecologic examination.
She reports regular menses that last for 4 days. She has no pelvic pain, pressure, or dyspareunia.
She has completed childbearing and her partner has had a vasectomy. On physical examination,
she has a palpable abdominal pelvic mass that extends to the midpoint between the umbilicus
and the pubic symphysis. Bimanual examination reveals a globally enlarged uterus the size of a
14-week gestation. Ultrasonography confirms multiple myomas and normal-appearing ovaries.
The best next step in her management is

(A) reassurance and follow-up in 1 year


(B) levonorgestrel IUD
(C) myomectomy
(D) endometrial ablation
(E) hysterectomy

73
A 55-year-old woman, gravida 2, para 2, comes to your office with vulvar pain for 8 months. She
has been treated repeatedly for yeast infections. She reports a constant raw feeling like sandpaper
on the vulva. The pain increases after intercourse. Your evaluation shows no visible abnormalities
of the vulva or vagina. Pain is present with separation of the labia minora. No tenderness is reported
with a cotton swab palpation of the vulva or vestibule. Pain mapping pelvic examination reveals no
areas of palpable tenderness. Microscopic evaluation of the vaginal discharge is unremarkable. She
is in good health and has no significant past medical history. The most likely diagnosis is

(A) chronic fungal infection


(B) lichen sclerosus
(C) bilateral pudendal neuralgia
(D) vulvodynia
(E) provoked vestibulodynia (vulvar vestibulitis)

74
A 16-year-old patient comes to your office with a 1-week history of increased vaginal discharge
and lower abdominal cramping. Her last menstrual period was 2 weeks ago. She reports being
sexually active, and says that she and her partner do not always use condoms. She has no fever,
vomiting, or diarrhea. She reports a possible mild reaction when given penicillin at age 4 years.
Pelvic examination reveals mucopurulent discharge and uterine tenderness. Microscopy reveals an
abundance of white blood cells, but yields negative results for trichomonas, monilia, and bacterial
vaginosis. A urine pregnancy test result is negative. The most appropriate antibiotic treatment for
this patient is

(A) outpatient azithromycin and metronidazole


(B) outpatient ceftriaxone and doxycycline
(C) inpatient clindamycin and gentamicin
(D) inpatient levofloxacin and doxycycline
(E) outpatient levofloxacin and metronidazole
28 PROLOG

75
A 30-year-old nulligravid woman has a colposcopic evaluation for a high-grade squamous intra-
epithelial lesion (HSIL) Pap test result. No ectocervical lesions are observed, and endocervical
curettage result is reported as negative. In light of these findings, the lesion that must be excluded is

(A) vulvar intraepithelial neoplasia 3


(B) adenocarcinoma in situ of the cervix
(C) vaginal intraepithelial neoplasia (VAIN) 3
(D) endometrial hyperplasia
(E) chronic cervicitis

76
A 53-year-old woman comes to your office for abnormal uterine bleeding. Her body mass index
is 35 (calculated as weight in kilograms divided by height in meters squared). Endometrial biopsy
is performed and the pathology report shows complex atypical endometrial hyperplasia and well-
differentiated endometrioid adenocarcinoma with squamous differentiation, International Federa-
tion of Gynecology and Obstetrics grade 1. During counseling, you explain to the patient that
the component of surgical staging for endometrial cancer that remains most controversial in this
setting is

(A) appendectomy
(B) bilateral salpingo-oophorectomy
(C) lymphadenectomy
(D) omentectomy
(E) random peritoneal biopsies

77
A 19-year-old woman comes to your office for insertion of an etonogestrel subdermal implant
for contraception. She reports that her menstrual cycles are regular and that her last menses were
5 days ago. Her body mass index is 28.4 (calculated as weight in kilograms divided by height
in meters squared). During counseling, you tell her that the most common adverse effect of the
etonogestrel subdermal implant is

(A) abnormal uterine bleeding


(B) headache
(C) weight gain
(D) breast pain
(E) emotional lability
Gynecology and Surgery 29

78
A 33-year-old woman, gravida 2, para 2, comes to your office for routine well-woman care. Her
medical history is significant for genital herpes (a genital lesion that was culture-positive for her-
pes simplex virus type 2 [HSV-2]) diagnosed 3 years ago. Since that time, she has noted recurrent
outbreaks every 1–2 months. She uses an oral antiviral drug as needed for outbreaks. Currently
she is asymptomatic. Your examination confirms normal vulvar anatomy without lesions. The best
management plan for this patient is

(A) confirmation of diagnosis with type-specific serologic test


(B) episodic antiviral therapy
(C) local antiviral therapy
(D) suppressive antiviral therapy

79
A 29-year-old woman, gravida 3, para 3, undergoes hysteroscopic sterilization in your office.
During the procedure, you perform paracervical block and cervical dilation. The right fallopian tube
is successfully occluded; however, the left fallopian tube cannot be cannulated because of spasm
despite repeated efforts. You decide to terminate the procedure and advise the patient to return in 2
weeks for hysteroscopic occlusion of the left fallopian tube. When you submit the billing for the ini-
tial procedure, in addition to the Current Procedural Terminology (CPT) code1 for hysteroscopic
tubal occlusion (58565), you will need to include a code for

(A) cervical dilation


(B) reduced service modifier
(C) paracervical block
(D) diagnostic hysteroscopy

80
A 38-year-old woman, gravida 2, para 2, comes to the emergency department with vaginal bleed-
ing and severe abdominal cramping. She underwent uterine artery embolization 14 weeks ago. Her
temperature is 38.5°C (101.3°F), blood pressure is 100 mm Hg systolic and 80 mm Hg diastolic,
and the heart rate is 104 beats per minute. Blood workup is significant for a white blood cell count
of 12,800 per microliter and hemoglobin level of 10.2 g/dL, decreased from a preoperative hemo-
globin level of 13.5 g/dL. The physician in the emergency department had difficulty performing
the speculum examination because of a mass in the vagina. Pelvic examination reveals uterine
tenderness but no adnexal tenderness. Gas is not present within the myoma as per the abdominal
and pelvic computed tomography (CT) scan. The most likely diagnosis is

(A) normal findings


(B) tubo–ovarian abscess
(C) pyomyoma
(D) prolapsed submucosal myoma
(E) uterine embolization material

1
Current Procedural Terminology (CPT) is copyright 2009 American Medical Association. All rights reserved. No fee schedules, basic
units, relative values, or related listings are included in CPT. The AMA assumes no liability for the data contained herein. Applicable
FARS/DFARS restrictions apply to government use. CPT® is a trademark of the American Medical Association.
30 PROLOG

81
A 14-year-old patient, who underwent menarche 15 months ago, has irregular, heavy menses that
occur every 2–8 weeks and last up to 14 days. She reports the bleeding to be heavy with clots, and
she is having accidents at school and at home. She has been experiencing menstrual cramps, but
reports that they are relieved by ibuprofen. Her blood pressure is 110 mm Hg systolic and 80 mm
Hg diastolic and the heart rate is 82 beats per minute. Her abdomen is soft and nontender; no masses
are noted. The remainder of the physical examination is unremarkable. An office urine pregnancy
test yields a negative result. The most likely etiology of her menstrual abnormality is

(A) chlamydial cervicitis


(B) hypothyroidism
(C) bleeding disorder
(D) anovulation
(E) endometriosis

82
A 37-year-old nulligravid woman is scheduled for an in-office diagnostic hysteroscopy for
abnormal uterine bleeding. During the procedure, the cervix is noted to be stenotic and dilation is
performed. The hysteroscope is then inserted with some force into the uterine cavity with saline
solution for distention. At first, distention is inadequate and the saline bag is raised higher on the
intravenous pole. A 2–3-mm defect is immediately noted at the top of the fundus with minimal
blood oozing into the cavity. No other endometrial pathology is noted. The procedure is terminated
and the fluid deficit is 150 mL. At the end of the procedure, the patient is pain free and her abdomen
is soft and nontender. She has a blood pressure of 117 mm Hg systolic and 82 mm Hg diastolic and
pulse of 72 beats per minute. The best next step in the management of this patient is

(A) observation
(B) immediate laparoscopy
(C) discharge home
(D) electrolyte measurement
(E) ultrasonography of abdomen and pelvis

83
A 32-year-old woman, gravida 3, para 3, who does not desire future fertility, has been experienc-
ing heavy menstrual bleeding. Her past medical history is significant for asthma; she is otherwise
healthy. The workup includes pelvic ultrasonography (normal result), hemoglobin measurement
(10.3 gm/dL), cervical screening for gonorrhea and chlamydia, and normal cervical cytology. Her
only sibling had been healthy until she recently received a diagnosis of pulmonary embolus at age
35 years. The most appropriate first-line therapy to treat heavy menstrual bleeding in this patient is

(A) combination oral contraceptives


(B) levonorgestrel intrauterine device (IUD)
(C) copper IUD
(D) endometrial ablation
(E) tranexamic acid
Gynecology and Surgery 31

84
A 16-year-old patient comes to the emergency department with lower abdominal pain for 48 hours.
She has no nausea, vomiting, diarrhea, or dysuria. She reports a normal appetite. Over the past year,
she has had three sexual partners. She is currently not using any birth control. Her last menstrual
period was 5 weeks ago. Physical examination shows a well-nourished young woman in no acute
distress. She has a mildly tender lower abdomen, left more than right, purulent discharge per cervi-
cal os, and left adnexal tenderness. Oral temperature is 38.1°C (100.5°F). Laboratory tests reveal a
total leukocyte count of 10,540 per microliter and a positive pregnancy test result. Ultrasonography
of the abdomen and pelvis yields a negative result for appendicitis or fluid in the cul-de-sac. The
contraindication to outpatient therapy in this patient is

(A) pregnancy
(B) leukocyte count
(C) body temperature
(D) her age
(E) left adnexal tenderness

85
A 54-year-old woman, gravida 3, para 2, comes to your office for evaluation of postmenopausal
bleeding. Approximately 6 months ago, she started continuous combined estrogen and progestin
therapy for postmenopausal symptoms. She states that her hot flushes and vaginal dryness have
improved since she began the hormone therapy (HT). She reports 1 week of light spotting and no
other symptoms. You schedule her for pelvic ultrasonography, which reveals an endometrial thick-
ness of 3 mm. The most appropriate management with respect to her medication is to

(A) stop the medication


(B) switch to cyclic progestin
(C) continue the same dose of HT
(D) add another form of estrogen
(E) increase the dose of HT

86
A 32-year-old immunocompetent woman is referred to you for further treatment of a genital con-
dyloma. She has been treated for genital condylomas (warts) by her family health care practitioner
initially using imiquimod followed by trichloroacetic acid for incomplete lesion resolution over
the past 6 months. Currently, the patient notes a continued “lump” on the mons pubis. Her vulva
appears as shown in Fig. 86-1 (see color plate). The next step in management is

(A) excisional biopsy


(B) laser ablation
(C) office cryotherapy
(D) re-treatment with imiquimod
(E) treatment with podophyllin resin
32 PROLOG

87
A 27-year-old woman, gravida 1, para 1, comes to your office at 8 weeks postpartum. Two weeks
previously, you diagnosed puerperal mastitis in this patient. At that time you prescribed dicloxacil-
lin and advised her to continue breastfeeding. Currently, she has a fever of 39°C (102°F) and you
observe a painful firm erythematous lump 2 cm in diameter in the lateral aspect of the right breast.
The most appropriate next step in management is

(A) ultrasonography-guided needle aspiration


(B) breast milk culture
(C) warm compress therapy
(D) change to different antibiotic therapy
(E) incision and drainage

88
A 34-year-old woman, gravida 3, para 3, comes to your office for a second opinion. She has had
three laparoscopic procedures for endometriosis-associated pelvic pain over the past 4 years. The
procedures have not helped to decrease her symptoms of dysmenorrhea, dyspareunia, and noncyclic
pelvic pain although all of the endometriotic tissue was removed each time. Her gynecologist has
recommended a hysterectomy and bilateral salpingo-oophorectomy (BSO). She does not desire
preservation of fertility. In the past year, she has been treated for six episodes of urinary tract
infections with symptoms of urgency, frequency, and pain. Culture was performed during her last
episode and showed negative results. She has no gastrointestinal symptoms. Bimanual examination
reveals severe, diffuse tenderness. The best next step in management is

(A) hysterectomy and BSO


(B) depot leuprolide treatment
(C) colonoscopy
(D) laparoscopy
(E) cystoscopy

89
An obese woman with type 2 diabetes mellitus is scheduled for a total abdominal hysterectomy for
large, symptomatic uterine myomas. She takes 40 units of long-acting insulin every evening and
1,000 mg of metformin daily. The preoperative hemoglobin A1c level is 6.8%. Hysterectomy takes
4 hours to perform and is complicated by an estimated blood loss of 750mL. On postoperative day
1, her vital signs are stable, and the urine output is 10 mL/hr. She is nauseated and is not eager to
resume oral intake. In addition to glucose-containing intravenous (IV) fluids, the best management
plan for postoperative glucose control in this patient is

(A) insulin regimen and metformin


(B) metformin alone
(C) long-acting insulin alone
(D) long-acting insulin plus sliding scale
(E) metformin with an insulin sliding scale
Gynecology and Surgery 33

90
A 44-year-old woman, gravida 3, para 3, has heavy, irregular menses and mild right lower quadrant
discomfort. The patient has declined hormonal management in the past because her mother has a
history of breast cancer. She is interested in endometrial ablation. She has a history of cesarean
delivery. On bimanual examination, she has mild right adnexal tenderness. Transvaginal ultraso-
nography shows a retroverted uterus with a sagittal length of 9.5 cm, endometrial echo of 6 mm, a
2-cm submucosal and fundal myoma, normal ovaries, and no free fluid. Endometrial biopsy reveals
disordered, proliferative phase endometrium and acute endometritis with the presence of plasma
cells. The absolute contraindication to endometrial ablation in this patient is

(A) history of cesarean delivery


(B) retroverted uterus
(C) submucosal myoma
(D) pelvic inflammatory disease
(E) uterine length greater than 8 cm

91
A 47-year-old woman, gravida 3, para 3, comes to your office with symptoms of stress urinary
incontinence. The patient undergoes placement of a tension-free vaginal tape with cystoscopy.
During the procedure, the patient is noted to have brisk bleeding from the trocar sites and becomes
hypotensive. The vessel that has been shown to have the closest mean distance to the lateral aspect
of the tension-free vaginal tape trocar is the

(A) superficial epigastric


(B) obturator
(C) inferior epigastric
(D) external iliac
(E) femoral

92
A 58-year-old woman has been treated for several months for chronic candidiasis despite negative
vaginal culture results. Your partner performed a vulvar biopsy 2 weeks ago that showed lichen
sclerosus. The patient has intense vulvar pruritus and moderate dyspareunia (numeric rating scale
pain score, 4 out of 10). On examination, you find the presence of significant adhesions at the cli-
toris that were not previously documented. Your next step in the treatment of this patient should be

(A) topical clobetasol propionate


(B) lysis of the clitoral adhesions
(C) triamcinolone hexacetonide injection
(D) repeat biopsy
(E) pimecrolimus cream
34 PROLOG

93
A 23-year-old woman, gravida 2, para 0, comes to the emergency department with spotting. The
time of her last menstrual period is unknown. Her history is significant for an ectopic pregnancy
2 years ago treated by laparoscopic salpingectomy. Examination reveals height 1.57 m (5 ft 2 in),
weight 61.2 kg (135 lb), blood pressure 112 mm Hg systolic and 65 mm Hg diastolic, pulse 72
beats per minute, and respiratory rate 16 breaths per minute. Her abdomen is soft and nontender.
The uterus is slightly enlarged, nontender with a palpable right adnexal mass. Laboratory results
reveal a quantitative human chorionic gonadotropin (hCG) level of 2,392 mIU/mL and hematocrit
of 36%; the blood type is O positive. Ultrasonography reveals an endometrial thickness of 17.8 mm
with no intrauterine gestational sac. Both ovaries are visualized, and a 3-cm complex mass in the
right ovary is identified. No other adnexal masses are noted. No fluid is present in the cul-de-sac.
The best next step in management is

(A) methotrexate sodium


(B) dilation and curettage
(C) diagnostic laparoscopy
(D) serum progesterone level
(E) hCG level measurement in 48 hours

94
A 51-year-old woman comes to your office for her annual gynecologic examination. Her history is
significant for hysterectomy for cervical intraepithelial neoplasia (CIN) 3 at age 45 years. Her Pap
test results have been normal since her hysterectomy, and her last normal Pap test was performed 1
year ago. In regard to the possible need for further screening, you advise her that she needs

(A) no further cytologic screening


(B) biennial thin-layer cytology with reflex human papillomavirus (HPV) testing
(C) thin-layer cytology and HPV cotesting every 5 years
(D) annual HPV testing
(E) annual thin-layer cytology
Gynecology and Surgery 35

95
A 60-year-old woman comes to your office for a preventive health visit. She reports no significant
interval medical history. Her family history includes her paternal grandmother with ovarian cancer
and a maternal cousin with postmenopausal breast cancer. The patient underwent a screening mam-
mography and colonoscopy within the last year, both with normal results. Breast, abdominal, and
rectal examinations yield normal results. A right adnexal mass is palpated on bimanual examina-
tion. Transvaginal ultrasonography shows the finding as shown in Figure 95-1. A CA 125 level is
11 units/mL and hemoglobin level is 12.7 g/dL. The best next step in the management is

(A) laparoscopy
(B) laparotomy
(C) repeat ultrasonography in 3–6 months
(D) gynecologic oncologist evaluation
(E) BRCA1 and BRCA2 testing

FIG. 95-1

96
A 21-year-old woman is undergoing a laparotomy for a cystectomy of a 20-cm teratoma on her
right ovary. Her medical history is otherwise unremarkable and her only medication is an oral
contraceptive. During the surgery, after lifting the right ovary off the right pelvic sidewall, the
anesthesiologist reports significant hypotension, tachycardia, hypoxemia, and decreased end-tidal
carbon dioxide. The most likely diagnosis is

(A) acute blood loss anemia


(B) pulmonary embolus
(C) acute myocardial infarction
(D) septic shock
36 PROLOG

97
A 66-year-old postmenopausal woman presents for follow-up after bone density screening. Her
T-score was –0.9. She takes daily vitamin D and calcium. Her past medical history is significant
for deep vein thrombosis after delivery of her second child 30 years ago. She currently reports
treatment for gastroesophageal reflux disease and osteoarthritis. She takes a daily antacid and acet-
aminophen as needed. She recently was treated with an oral steroid taper for 10 days because of an
urticarial skin reaction. The best recommendation to maintain her bone mineral density is

(A) oral estrogen and progestin


(B) selective estrogen receptor modulators
(C) bisphosphonates
(D) calcium-rich foods and strength-building exercise

98
A 29-year-old woman is referred to you for reproductive health counseling. She was treated with
a total colectomy at age 19 years for right-sided colon cancer. The cancer was associated with an
MLH1 mismatch repair mutation by immunohistochemistry studies. She is adamant that she does
not want to get pregnant. The most appropriate risk-reducing strategy is

(A) progestin-containing intrauterine device


(B) permanent tubal occlusion
(C) vaginal hysterectomy
(D) hysterectomy with bilateral salpingo-oophorectomy (BSO)
(E) BSO

99
A 44-year-old woman with symptomatic myomas comes to your office for a preoperative evalua-
tion. She is not at an increased genetic risk of ovarian cancer. As part of the hysterectomy informed
consent, you discuss the risks and benefits of ovary retention at the time of surgery. You explain
that, compared with the general population, if she has a hysterectomy with ovarian retention, the
time at which she is likely to experience menopause is

(A) at a similar age


(B) at an older age
(C) immediately postoperatively
(D) at a younger age
Gynecology and Surgery 37

100
A 55-year-old multiparous woman is evaluated for urinary incontinence. She underwent a trans-
obturator tape procedure 3 years ago for the clinical diagnosis of stress urinary incontinence.
The patient reports some incontinence with activity and Valsalva maneuvers but no urgency.
Urodynamic testing is significant for a maximal urethral closing pressure of less than 40 cm H2O.
The most likely diagnosis for urinary incontinence in this patient is

(A) overactive bladder


(B) vesicovaginal fistula
(C) intrinsic sphincter deficiency
(D) overflow incontinence
(E) recurrent stress incontinence

101
A 45-year-old woman with a body mass index of 24 (calculated as weight in kilograms divided
by height in meters squared) undergoes a total abdominal hysterectomy for asymptomatic uterine
fibroids. She receives intravenous cefazolin 1 hour before skin incision. The procedure is compli-
cated by extensive adhesiolysis, repair of a small bowel injury, and cystotomy. Estimated blood
loss is 700 mL, and the procedure is completed in 4.5 hours. The patient is extubated without
difficulty and transferred to the recovery area. During quality improvement review, the patient is
noted to have received inadequate prophylactic antibiotics during the procedure. The most likely
source of the inadequacy was

(A) lack of antibiotic administration at the time of cystotomy repair


(B) timing of preoperative antibiotics
(C) length of surgical procedure
(D) amount of estimated blood loss

102
A 46-year-old woman has been treated with breast-conserving surgery and radiotherapy for ductal
carcinoma in situ of the breast. Her medical oncologist recommends that she take adjuvant tamoxi-
fen citrate therapy. The benefit of tamoxifen therapy for this patient is that it will decrease her risk
of

(A) endometrial cancer


(B) death from breast cancer
(C) ipsilateral recurrent ductal carcinoma in situ (DCIS)
(D) invasive breast cancer
(E) ovarian cancer
38 PROLOG

103
A 48-year-old woman comes to your office for a preventive health care visit. She expresses frustra-
tion about her weight and her difficulty in losing weight. Her body mass index (BMI) is 40 (calcu-
lated as weight in kilograms divided by height in meters squared). The serum thyroid-stimulating
hormone level is normal. You recommend that in addition to exercise, the most effective dietary
plan to lose weight is to maintain a diet that is

(A) low in fat


(B) low in carbohydrates
(C) high in proteins
(D) calorie-reduced

104
A 55-year-old woman, gravida 3, para 3, with type 2 diabetes mellitus and a body mass index of
35 (calculated as weight in kilograms divided by height in meters squared), comes to your office
for her annual well-woman examination. At that time, a Pap test is completed and the result is
remarkable for atypical glandular cells. She also reports intermittent vaginal bleeding. An office
endometrial biopsy demonstrates complex atypical hyperplasia. The most appropriate next step is

(A) pelvic ultrasonography


(B) progestin therapy
(C) levonorgestrel intrauterine device
(D) dilation and curettage
(E) hysterectomy

105
A 38-year-old woman reports bilateral breast tenderness and heaviness. Her discomfort is primar-
ily in the lateral aspects of both breasts. She has no family history of breast cancer, is otherwise
healthy, and takes no medications. She has not detected a breast mass nor has she noted any nipple
discharge or skin changes. The breast and axillary examinations yield normal results with some
generalized fullness and lumpiness in the upper outer quadrants of both breasts. The most appro-
priate next step is

(A) reassurance
(B) breast magnetic resonance imaging
(C) bilateral mammography
(D) low-dose combined oral contraceptives
(E) tamoxifen citrate
Gynecology and Surgery 39

106
At her first gynecologic visit, a 20-year-old healthy woman requests combination oral contracep-
tives. She has irregular cycle intervals of 21–34 days. Her menstrual periods last up to 5 days and
are characterized by heavy blood flow. She recently became sexually active. Her family history is
unremarkable except for her mother who had a confirmed diagnosis of deep vein thrombosis. You
advise her that before starting oral contraceptives she should undergo

(A) an endometrial biopsy


(B) cervical cytology screening
(C) a factor V Leiden test
(D) pelvic ultrasonography
(E) serum testing for thyroid-stimulating hormone

107
A newly-married 26-year-old woman tells you that her husband informed her he previously
received a diagnosis of genital herpes but has been asymptomatic for several years. She has never
had any lesions. Her pelvic examination is normal. The most appropriate recommendation in the
management of this patient is that she

(A) start suppressive therapy


(B) recommend her partner start suppressive therapy
(C) undergo type-specific serologic testing
(D) consider suppressive therapy in pregnancy
(E) undergo serologic testing only if she becomes pregnant

108
Two weeks after an uncomplicated total abdominal hysterectomy for uterine myomas, a 48-year-
old woman comes to your office and reports frequent urination. She reports that she has “leaking
day and night,” and that it is worse in the morning. The problem started 2 days ago. Pelvic exami-
nation reveals clear fluid at the vaginal introitus. Urinalysis result is significant for microscopic
hematuria. The most likely diagnosis is

(A) postoperative urinary tract infection


(B) vesicovaginal fistula
(C) stress urinary incontinence
(D) overactive bladder syndrome
40 PROLOG

109
A 75-year-old woman, gravida 2, para 2, with an unremarkable medical history underwent an
uncomplicated total abdominal hysterectomy and bilateral salpingo-oophorectomy. On postopera-
tive day 2, you are contacted by a nurse regarding this patient’s new onset confusion. On evalua-
tion, the patient is in moderate distress and her vital signs are significant for elevated blood pressure
of 140 mm Hg systolic and 90 mm Hg diastolic and a heart rate of 80 beats per minute. She is
alert and oriented to herself but not to place or time. Examination yields unremarkable results with
clear breath sounds, normal heart rate, and positive bowel sounds with appropriate incision tender-
ness. Electrocardiography performed at the bedside produces unchanged results from those of her
preoperative electrocardiography performed 2 weeks before surgery. Complete blood count and
metabolic electrolytes levels are in the normal range. The best next step in her care is to

(A) order complete cardiac workup


(B) review her current medications to limit sedating drugs
(C) order a computed tomography of the head
(D) order neurology consultation

110
A 19-year-old nulligravid patient presents for a follow-up appointment after levonorgestrel intra-
uterine device (IUD) placement 6 weeks ago. During the speculum examination, the strings from
the system are not visible and cannot be teased from the cervix with a cytobrush. Ultrasonography
is performed and does not demonstrate the levonorgestrel IUD in the endometrial cavity. The most
appropriate next step is to perform

(A) pelvic magnetic resonance imaging


(B) abdominal–pelvic computed tomography
(C) diagnostic laparoscopy
(D) dilation and curettage and hysteroscopy
(E) radiography of the abdomen

111
You are completing the abdominal closure after a cesarean delivery in a multigravid woman with
an antenatal course complicated by smoking and gestational diabetes mellitus. Her body mass index
is 40 (calculated as weight in kilograms divided by height in meters squared). The thickness of the
subcutaneous adipose tissue is 6 cm. The step most likely to prevent separation of the skin and
superficial tissues in the setting of cesarean delivery is

(A) closure of the subcutaneous tissue


(B) placement of a closed subcutaneous drain
(C) placement of pressure dressing
(D) placement of surgical staples
Gynecology and Surgery 41

112
An 18-year-old patient asks you for a prescription for levonorgestrel-containing emergency con-
traception. Her last menstrual period was 5 weeks ago and she has a history of irregular menses.
Recently, she received a diagnosis of chlamydial infection and has not completed her prescribed
antibiotics. She has missed several of her progestin-only contraceptive pills during the current
cycle. The urine pregnancy test yields a positive result. Her body mass index is 32 (calculated as
weight in kilograms divided by height in meters squared), and she is known to have a heterozy-
gous factor V Leiden mutation. You advise her that a contraindication to prescribing emergency
contraception for her is

(A) current antibiotic use


(B) poor compliance
(C) positive urine pregnancy test result
(D) heterozygosity for factor V Leiden mutation
(E) obesity

113
A 42-year-old woman, gravida 1, para 1, reports fatigue and menstrual cycles with heavy menstrual
bleeding. Her menstrual interval is every 30 days with 4 days of heavy bleeding; she frequently
soils her clothes due to the heavy flow. She takes a nonsteroidal antiinflammatory drug during the
menses for painful cramping. She has been prescribed a daily diuretic for management of hyperten-
sion but has difficulty remembering to take it. On physical examination, her height is 1.6 m (64 in);
weight, 105.2 kg (140 lb); blood pressure, 150 mm Hg systolic and 95 mm Hg diastolic; and pulse,
72 beats per minute. The pelvic examination is notable for a small anteverted uterus and no pelvic
masses. The hemoglobin level is 9.5 g/dL and thyroid-stimulating hormone level is 2.5 mIU/L.
Endometrial biopsy and pelvic ultrasonography yield normal results, and she has no personal or
family history of excessive bruising or epistaxis. The patient declines the use of the levonorgestrel
intrauterine device for the management of heavy bleeding because she is fearful of a foreign body
inside her. The best medical option for this patient is

(A) transdermal contraceptive patch


(B) oral contraceptives
(C) tranexamic acid
(D) progestin-only minipill
42 PROLOG

114
A 36-year-old woman comes to your office for an initial prenatal visit. She states that her last men-
strual period occurred 10 weeks ago. She does not report any bleeding or problems with nausea and
vomiting. On bimanual examination, you note a small uterus. Ultrasonography confirms an empty
gestational sac consistent with an embryonic pregnancy loss. You counsel her regarding the risks
and benefits of medical versus surgical treatment of pregnancy loss as they pertain to blood loss,
infection, treatment follow-up, and patient accessibility. You inform her that use of misoprostol is
advantageous to surgical evacuation because it

(A) provides easier patient access


(B) leads to decreased blood loss
(C) has a decreased risk of infection
(D) requires no follow-up

115
A 29-year old woman presents to your office following treatment for a fourth urinary tract infec-
tion (UTI) after sexual intercourse in the past year. She has a test-of-cure result that indicates that
her last course of therapy was successful and she is asymptomatic today. You discuss with her a
number of approaches to reduce the risk of such infections, including prophylactic antibiotics, daily
ingestion of cranberry juice, voiding immediately after intercourse, postcoital douching, and daily
ingestion of D-mannose. She would prefer to avoid antibiotics because antibiotic therapy for her
UTIs has led to monilial vaginitis on two occasions. You inform her that the method with the best
evidence to support reduction in risk of recurrent urinary tract infections is

(A) prophylactic antibiotics


(B) daily cranberry juice
(C) postcoital voiding
(D) postcoital douching
(E) daily D-mannose

116
A 17-year-old patient has had dysmenorrhea for the past 2 years. In the first year after menarche,
the dysmenorrhea was successfully managed with over-the-counter pain medications and heat-
ing pads. Subsequently, the dysmenorrhea worsened and was characterized by nausea, vomiting,
and missed days from school. Despite treatment with extended-cycle oral contraceptive pills and
prescription-strength nonsteroidal antiinflammatory drugs (NSAIDs), school absences consistently
occurred with menses. She has no bowel or bladder problems and has never been sexually active.
Pelvic ultrasonography yields normal results. The most appropriate next step is

(A) dilation and curettage with hysteroscopy


(B) pelvic magnetic resonance imaging (MRI)
(C) diagnostic laparoscopy
(D) leuprolide acetate
(E) empiric broad-spectrum antibiotics
Gynecology and Surgery 43

117
A 42-year-old woman, gravida 2, para 2, with an unremarkable past medical history recently
received a diagnosis of a BRCA 1 mutation. She is in good health and has a body mass index of
28 (calculated as weight in kilograms divided by height in meters squared). Her mother received a
diagnosis of stage IIIC ovarian cancer at age 56 years. The best option to reduce the future risk of
ovarian and breast cancer in this patient is

(A) combined estrogen–progesterone oral contraceptive (OC) use until age 50 years
(B) prophylactic mastectomy and bilateral salpingo-oophorectomy
(C) biannual transvaginal ultrasonography, breast magnetic resonance imaging, and
mammography
(D) bilateral tubal ligation

118
A 39-year-old woman, gravida 2, para 2, comes to the clinic and requests contraception. She has
been in a monogamous relationship with a male sexual partner for the past 10 months and is not
using any consistent contraceptive method. She reports a history of irregular menses that occur
every 35–56 days without any intermenstrual spotting. On physical examination, her body mass
index (BMI) is 33 (calculated as weight in kilograms divided by height in meters squared), blood
pressure is 135 mm Hg systolic and 82 mm Hg diastolic, and pulse is 76 beats per minute. The
abdominal and pelvic examinations are difficult to perform secondary to her body habitus, but no
masses or point tenderness is noted. Urine pregnancy test and cultures for Neisseria gonorrhoeae
and Chlamydia trachomatis yield negative results. You advise the patient that the best contracep-
tive option for her is

(A) combination oral contraceptive


(B) combination transdermal contraceptive patch
(C) progestin-only contraceptive
(D) levonorgestrel IUD

119
A 26-year-old nulligravid woman reports increased frequency and pain with urination for 3 days.
In the past 3 years, she has called your office multiple times to report similar symptoms. Pelvic
examination yields normal results. The prior urine culture result was positive for Escherichia coli.
The current urine analysis yields positive results for nitrates and leukocytes esterase and negative
results for blood. You treat the patient with antibiotics and send her urine for culture and sensitivity
testing. After completion of the course of antibiotics, the best next step in management is to

(A) refer the patient to a urologist


(B) perform a test of cure in 1–2 weeks
(C) perform intravenous pyelography
(D) perform renal ultrasonography
(E) start daily prophylactic antibiotics
44 PROLOG

120
A 54-year-old woman is seeking therapy for worsening menopausal symptoms. The patient reports
hot flushes, night sweats, vaginal dryness, breast tenderness, joint stiffness, and general aches and
pains. She has been postmenopausal for 2 years and states that her symptoms are starting to affect
her performance at work and her relationships at home. She is in good health and is not currently
taking any medications. Her family history is negative for breast cancer and clotting disorders.
The patient is interested in starting hormone therapy (HT). Of the symptoms described, HT is least
likely to improve her

(A) joint stiffness


(B) vaginal dryness
(C) breast tenderness
(D) general aches and pains
(E) night sweats

121
A 45-year-old woman, gravida 2, para 2, with an unremarkable medical history underwent an
uncomplicated total vaginal hysterectomy for adenomyosis. She has a body mass index of 22
(calculated as weight in kilograms divided by height in meters squared). On the night after the
surgery, you are contacted by a nurse who reports new onset abdominal pain and oliguria in
this patient. On evaluation, the patient is in moderate distress and her vital signs are significant
for blood pressure of 140 mm Hg systolic and 90 mm Hg diastolic and a heart rate of 90 beats
per minute. Examination reveals moderate suprapubic tenderness with abdominal distention and
approximately 50 mL of clear urine in the Foley bag over the past 4 hours. The best next step in
management is

(A) complete blood count


(B) intravenous bolus of 1,000 mL of isotonic fluids
(C) computed tomography of abdomen and pelvis
(D) flushing the Foley catheter

122
A 19-year-old female college athlete is considering trying extended-cycle combined oral contra-
ceptives (OCs) for birth control and menstrual suppression. She has recently become sexually
active and has used only condoms. You counsel her that the use of the extended-cycle formulation
compared with the standard formulations will increase her risk of

(A) contraceptive failure


(B) breast cancer
(C) deep vein thrombosis
(D) weight gain
(E) unscheduled bleeding
Gynecology and Surgery 45

123
A 46-year-old woman, gravida 2, para 2, with type 2 diabetes mellitus comes to your office with
recurrent itching and vaginal discharge. A wet preparation result confirms yeast. This is the fourth
time she has had a confirmed vaginal yeast infection in the past 6 months. She received the diag-
nosis of diabetes mellitus 4 years ago and maintains her hemoglobin A1c level at 5.1–5.5 g/dL. The
most effective strategy to prevent recurrent yeast infections in this patient is

(A) suppressive antifungal therapy


(B) treatment of her sexual partner
(C) improved glycemic control
(D) oral antihistamine

124
An 18-year-old woman, gravida 1, para 0, presented to your office with an unplanned pregnancy at
8 weeks of gestation. The patient opted to undergo pregnancy termination and stated that she does
not want to become pregnant again anytime soon. In the recovery room, she is counseled regard-
ing contraceptive options and refuses an implantable progestin rod. The best next step to prevent a
repeat unplanned pregnancy is to

(A) prescribe oral contraceptives


(B) administer depot medroxyprogesterone acetate
(C) recommend condom use
(D) schedule insertion of an intrauterine device (IUD)
(E) prescribe the vaginal contraceptive ring

125
A 28-year-old nulligravid woman with tubal factor infertility is preparing for her first cycle of in
vitro fertilization (IVF). She was treated for pelvic inflammatory disease 9 years ago, and recent
transvaginal ultrasonography suggests bilateral hydrosalpinges. She states that she does not have
any abdominal pain, and her examination does not reveal adnexal tenderness. The most appropriate
next step in management is

(A) reassurance
(B) laparoscopic bilateral salpingectomy
(C) broad spectrum oral antibiotics
(D) laparoscopy with drainage of hydrosalpinges
46 PROLOG

126
A 70-year-old woman underwent an uncomplicated total laparoscopic hysterectomy and bilateral
salpingo-oophorectomy for complex endometrial hyperplasia with atypia. Her medical history
is remarkable for hypertension that requires multiple medications, including a calcium channel
blocker, a b-blocker, and an angiotensin-converting enzyme (ACE) inhibitor. You are concerned
about her risk of a postoperative myocardial infarction. The hypertension medication in the periop-
erative period that is most likely to help reduce her risk for postoperative myocardial infarction is

(A) b-blocker
(B) ACE inhibitor
(C) a-blocker
(D) calcium channel blocker

127
A 36-year-old woman, gravida 1, para 1, comes to the office for counseling regarding ovarian
cancer risk reduction. She has a history of receiving four cycles of clomiphene citrate for ovula-
tion induction to conceive. She tells you that her sister received a diagnosis of breast cancer at age
33 years, and that she and her sister have a BRCA 1 mutation. The intervention that has the highest
likelihood for reducing her risk for ovarian cancer is

(A) tamoxifen citrate


(B) oral contraceptives
(C) prophylactic bilateral salpingo-oophorectomy (BSO)
(D) tubal ligation

128
A 53-year-old woman, gravida 2, para 2, informs you that her last menstrual period was approxi-
mately 13 months ago and that since that time she has had significant hot flushes and sleep
disturbance. She states that she wakes up several times a night sweating. Her medical history is
significant for breast cancer diagnosed 3 years ago, for which she takes tamoxifen citrate. The best
initial treatment for this patient is

(A) black cohosh


(B) ginseng
(C) venlafaxine
(D) combined estrogen and progestin therapy
(E) gabapentin
Gynecology and Surgery 47

129
A 54-year-old postmenopausal woman, gravida 2, para 2, comes to your office for her well-woman
examination. Her clinical breast examination is normal, but screening mammography shows an
abnormal result. Her past medical history is significant for obesity and diabetes mellitus. Her father
had colon cancer at age 71 years and her mother died of a heart attack at age 59 years. Diagnostic
mammography and fine needle aspiration show ductal carcinoma in situ (DCIS) in the left breast.
The best initial treatment option is

(A) local surgical treatment


(B) local radiation therapy
(C) tamoxifen citrate
(D) aromatase inhibitor

130
A 21-year-old nulligravid woman comes to your office with menstrual irregularity. She has not had
a menstrual cycle for 6 months. Her menses have been irregular since menarche at age 16 years.
She has no medical problems and exercises daily. She has been a competitive cross-country runner
since age 13 years. She keeps a strict vegan diet because she does not want to gain weight as many
of her classmates have since they began college. Her body mass index is 17 (calculated as weight
in kilograms divided by height in meters squared), blood pressure is 90 mm Hg systolic and 55 mm
Hg diastolic, and pulse is 50 beats per minute. She is not interested in childbearing at this time. In
addition to counseling about her menstrual irregularity, you tell her that her most significant health
problem that needs to be addressed at this time is risk of

(A) dental caries


(B) osteoporosis
(C) ruptured ovarian cysts
(D) facial hair growth

131
A 25-year-old woman, gravida 1, para 0, at 22 weeks of gestation comes to the emergency depart-
ment after cutting her hand with a gardening tool. It has been 6 years since the patient’s last tetanus
shot, and she does not believe she has ever received vaccination for tetanus, diphtheria, and pertus-
sis (Tdap). The wound is irrigated and sutured. The most appropriate next step in her management
is to administer

(A) one dose of tetanus and diphtheria vaccine


(B) one dose of Tdap vaccine postpartum
(C) one dose of Tdap vaccine immediately
(D) three-part series of Tdap vaccine
(E) one dose of tetanus and diphtheria vaccine today and pertussis vaccination
postpartum
48 PROLOG

132
A 27-year-old woman comes to the emergency department with vaginal bleeding and pain. Her
last menstrual period was 19 weeks ago and she reports delivering fetal tissue at home. Her vital
signs are as follows: temperature, 38.9°C (102°F); pulse, 110 beats per minute; and blood pressure,
98 mm Hg systolic and 59 mm Hg diastolic. She is bleeding significantly. On bimanual examina-
tion, her uterus is tender and enlarged, the cervix is dilated 2 cm, the hematocrit is 25%, and the
white blood cell count is 21,000 cells per microliter. Transvaginal ultrasonography reveals hetero-
geneous material in the uterus. In addition to initiating broad-spectrum intravenous antibiotics, the
best option for her management is

(A) misoprostol
(B) dilation and evacuation
(C) blood transfusion
(D) diagnostic laparoscopy

133
All adjuvant treatment was unsuccessful in an 82-year-old woman with a history of recurrent
ovarian cancer. She agreed to hospice care after discussion with her oncologist last month but is
admitted to the emergency department tonight with new onset nausea and vomiting. She remains
mentally alert and can make decisions about her health. A computed tomography result is consis-
tent with a multi-focal bowel obstruction indicative of progressive disease. You are contacted by
the patient’s daughter who wants everything possible done for her mother’s care. The most reason-
able next step in her care is

(A) schedule for surgery


(B) start total parenteral nutrition
(C) reassure the daughter outside patient room
(D) schedule a meeting with the patient and family

134
A 33-year-old woman, gravida 2, para 2, comes to your office for contraception counseling. She
underwent gastric bypass surgery 18 months ago and lost 45.4 kg (100 lb). Her current weight is
100.7 kg (222 lb). Her height is 1.75 m (69 in), and she has a body mass index of 32.7 (calculated
as weight in kilograms divided by height in meters squared). She has a new partner and is interested
in birth control options. She adheres to the weight management program and takes multivitamins
daily. Her past medical history is otherwise unremarkable. The most suitable contraceptive option
for this patient is

(A) combined oral contraceptives


(B) oral progestin-only contraceptives
(C) transdermal contraceptive patch
(D) diaphragm and contraceptive foam
(E) levonorgestrel IUD
Gynecology and Surgery 49

DIRECTIONS: Each of the questions below consists of lettered headings followed by a list of
numbered words or statements. For each numbered word or statement, select the ONE lettered
heading that is associated most closely with it and fill in the circle containing the corresponding
letter on the answer sheet. Each lettered heading or lettered component may be selected once,
more than once, or not at all.

135–137
For each scenario (135–137), select the patient safety approach (A–E) that would be the most cost-
effective in addressing the described problem.

(A) A fully stocked code cart


(B) An emergency kit
(C) Simulation training
(D) An on-site anesthesiologist
(E) Personnel certified in advanced cardiac life support available on site

135. Your practice is planning to institute office-based endometrial ablation procedures using
conscious sedation.
136. An elderly patient who recently changed blood pressure medications faints in the waiting
room. Transfer to a nearby emergency department does not go smoothly.
137. At the end of a loop electrosurgical excision procedure, a patient has a severe vasovagal
reaction.

138–142
For the patient in each clinical scenario (138–142), match the best cervical cancer screening option
(A–C).

(A) No screening recommended at this visit


(B) Cervical cytology with or without high-risk human papillomavirus testing.
(C) No further cervical cytology screening recommended.

138. A 26-year-old woman with a normal Pap test result 1 year ago
139. A 19-year-old woman who initiated vaginal intercourse 2 years ago
140. A 35-year-old morbidly obese female with diabetes mellitus and a normal Pap test result
3 years ago
141. A 46-year-old woman after total laparoscopic hysterectomy for stage 4 endometriosis with
normal Pap test history
142. A 72-year-old postmenopausal woman with stress urinary incontinence and normal Pap test
history
50 PROLOG

143–146
For the patient in each scenario (143–146), choose the likely diagnosis (A–E).

(A) Injury of a patent urachus


(B) Laceration of an inferior epigastric artery
(C) Urinary sepsis
(D) Ilioinguinal nerve entrapment
(E) Occult injury of adherent bowel at the umbilicus

143. Twelve hours after she has undergone a laparoscopic hysterectomy, a patient with a history
of prior cesarean delivery develops fever of 40°C (104°F), emesis, and abdominal rigidity
and rebound with hyperactive bowel sounds.
144. Two days after laparoscopic Falope ring sterilization, a patient receives the diagnosis of
urinary ascites. A midline suprapubic trocar–cannula site was used for ring application.
145. After insertion of a 12-mm trocar–cannula in the right lower abdomen, the patient has sig-
nificant bleeding from the cannula.
146. Postoperatively, a patient experiences excruciating pain at the site of a 12-mm right lower
quadrant cannula used for uterine morcellation. On examination, severe allodynia is
observed to light touch medial to the incision.

147–149
For the patient in each scenario (147–149), choose the likely diagnosis (A–E).

(A) BRCA 1 gene mutation


(B) BRCA 2 gene mutation
(C) Lynch II syndrome
(D) Li–Fraumeni syndrome
(E) Cowden disease

147. A 48-year-old woman with breast cancer has a family history of breast cancer in her mother,
her maternal grandmother, and her younger sister
148. A 48-year-old woman with advanced ovarian cancer has a family history of breast cancer in
her mother and ovarian cancer in a maternal aunt
149. A 48-year-old woman with ovarian cancer has a family history of colon cancer in her father
and brother
Gynecology and Surgery 51

150–153
For the patient in each scenario (150–153), choose the most appropriate tumor marker (A–F).

(A) CA 125
(B) Alpha-fetoprotein (AFP)
(C) Inhibin B
(D) Follicle-stimulating hormone
(E) Carcinoembryonic antigen (CEA)
(F) Beta subunit of human chorionic gonadotropin (b-hCG)

150. A 16-year-old girl with a unilateral solid adnexal mass and regular menses
151. A 50-year-old woman with severe abdominal pain, a 14-cm unilateral complex adnexal mass,
and hemoperitoneum
152. A 28-year-old woman at 5 months postpartum with bilateral 6-cm solid adnexal masses
153. A 50-year-old woman with pelvic pressure and a unilateral 8-cm cystic adnexal mass with
mural nodularity

154–158
For the patient in each scenario (154–158), choose the likely diagnosis (A–E).

(A) Lichen sclerosus


(B) Streptococcal vaginitis
(C) Nonspecific vaginitis
(D) Physiologic discharge
(E) Retained foreign object

154. A 4-year-old girl has a 6-month history of intermittent dysuria, vulvar pruritus, and vagi-
nal discharge. Physical examination reveals erythema of the vestibule and perineum and a
yellow, foul-smelling discharge.
155. A 6-year-old girl has a persistent green, foul-smelling discharge often tinged with blood.
Physical examination yields a normal result except for a thick green discharge noted at introi-
tus.
156. A 7-year-old girl has constant vulvar pruritis, dysuria, and soreness. Physical examina-
tion shows skin changes in a figure-of-eight pattern around the vagina and anus with white
sharply demarcated skin with fine wrinkles, fissures, and blood blisters.
157. A 9-year-old girl has thick, pasty clear to white vaginal discharge. The physical examination
reveals Tanner 3 breast development and a normal perineum and introitus.
158. A 5-year-old girl has acute onset of green vaginal discharge. She has fever, chills, and ear
pain. The physical examination reveals a green discharge and an erythematous introitus.
52 PROLOG

159–162
For the patient in each scenario (159–162), indicate the appropriate human papillomavirus (HPV)
vaccination (A–C).

(A) Human papillomavirus vaccine not recommended


(B) Full (three-dose) course of the HPV vaccine
(C) One dose of HPV vaccine

159. A 42-year-old woman with a new sexual partner and a history of normal Pap test results
160. A 12-year-old girl who is not sexually active
161. A 14-year-old boy who is sexually active with a history of two lifetime partners
162. A 19-year-old woman who received the scheduled first two doses of HPV vaccination
2 years ago

163–165
For the patient in each scenario (163–165), choose the best treatment option (A–E).

(A) Pelvic floor physical therapy


(B) Vestibulectomy
(C) Trigger point injection
(D) Oral anticonvulsant
(E) Local corticosteroid

163. A 27-year-old nulligravid woman has localized vestibulodynia that has not responded to pre-
vious therapy that included oral tricyclic antidepressant, oral antiepileptic, topical lidocaine,
compounded topical antiepileptic, and cognitive behavioral counseling.
164. A 20-year-old nulligravid woman with vaginismus requests treatment to facilitate tampon use
for swimming during menstruation. Ultimately, she desires vaginal intercourse.
165. A 51-year-old nulligravid woman has generalized vulvodynia unresponsive to local skin care
guidelines, trial of topical anesthetic, and prescribed cognitive behavioral counseling. Also,
she notes night sweats and sleep disruption.
Gynecology and Surgery 53

PROLOG USER DEMOGRAPHICS AND PROLOG EVALUATION

To obtain a profile of the group of Fellows choosing to use self-assessment as a continuing medi-
cal education tool, the Education Division would appreciate the following information. This is for
education program planning only and will not be associated with score reports in any way.

Part I. Demographics
166. Which of the following best describes the focus of your practice?
(A) General Obstetrics and Gynecology
(B) Oncology
(C) Maternal–Fetal Medicine
(D) Endocrinology
(E) Urogynecology
(F) Other
167. Are you board certified in obstetrics/gynecology?
(A) Yes
(B) No
168. Do you have subspecialty certification?
(A) Yes
(B) No
169. Have you been recertified?
(A) Yes
(B) No
170. How many years have you been in practice?
(A) 10 years or less
(B) 11–20 years
(C) More than 20 years

Part II. PROLOG Evaluation


For each of the reasons listed (171–174), indicate the degree of importance to you for selecting
PROLOG as a means of professional development, using the scale below:
4 = Very important
3 = Somewhat important
2 = Not very important
1 = Not important at all
171. Update my knowledge.
172. Verify my current practice.
173. Earn continuing medical education credits.
174. Use as a home study program.
For questions 175–179, indicate whether you are using this unit of PROLOG to prepare for the
following:
175. The in-training examination of the Council on Residency Education in Obstetrics and
Gynecology (CREOG)
176. The written board examination of the American Board of Obstetrics and Gynecology
(ABOG)
54 PROLOG

177. The oral board examination of ABOG


178. The 6-year recertification examination of ABOG
179. In your estimation, what percentage of the content in PROLOG is new information to you?
(A) Less than 10%
(B) 10–20%
(C) 21–50%
(D) 51–75%
(E) More than 75%
For each of the statements below (180–184), indicate the degree to which you agree, using the
following scale:
4 = Strongly agree
3 = Somewhat agree
2 = Somewhat disagree
1 = Strongly disagree
180. The items in PROLOG closely relate to my clinical practice.
181. PROLOG is helpful in assessing my knowledge.
182. I have made one or more changes in patient management after reading PROLOG.
183. The critiques are useful.
184. The references accompanying each answer are useful.
Indicate how you would rate the following (185–196) in preference for obtaining professional
development, using the following scale:
4 = Highest preference
3 = Somewhat high
2 = Somewhat low
1 = Lowest preference
185. Postgraduate courses
186. ACOG district annual meeting
187. ACOG Annual Clinical Meeting
188. PROLOG self-assessment
189. PROLOG Dialogue audio programs
190. Videotape programs
191. CME programs offered by hospital departments
192. CME programs offered by other organizations
193. CD-ROM
194. Internet
195. Was experimental or unlabeled use of products clearly reflected in the critiques?
Yes
No
196. Was there evidence of “conflict of interest” disclosures by task force members?
Yes
No
Gynecology and Surgery 55

Appendix A
Normal Values for Laboratory Tests*

Analyte Conventional Units


Alanine aminotransferase, serum 8–35 units/L
Alkaline phosphatase, serum 15–120 units/L
Menopause
Amniotic fluid index 3–30 mL
Amylase 20–300 units/L
Greater than 60 years old 21–160 units/L
Aspartate aminotransferase, serum 15–30 units/L
Bicarbonate
Arterial blood 21–27 mEq/L
Venous plasma 23–29 mEq/L
Bilirubin
Total 0.3–1 mg/dL
Conjugated (direct) 0.1–0.4 mg/dL
Newborn, total 1–10 mg/dL
Blood gases (arterial) and pulmonary function
Base deficit Less than 3 mEq/L
Base excess, arterial blood, calculated –2 mEq/L to +3 mEq/L
Forced expiratory volume (FEV1) 3.5–5 L
Greater than 80% of predicted value
Forced vital capacity 3.5–5 L
Oxygen saturation (So2) 95% or higher
Pao2 80 mm Hg or more
Pco2 35–45 mm Hg
Po2 80–95 mm Hg
Peak expiratory flow rate Approximately 450 L/min
pH 7.35–7.45
Pvo2 30–40 mm Hg
Blood urea nitrogen
Adult 7–18 mg/dL
Greater than 60 years old 8–20 mg/dL
CA 125 Less than 34 units/mL
Calcium
Ionized 4.6–5.3 mg/dL
Serum 8.6–10 mg/dL
Chloride 98–106 mEq/L
Cholesterol
Total
Desirable 140–199 mg/dL
Borderline high 200–239 mg/dL
High 240 mg/dL or more
High-density lipoprotein 40–85 mg/dL
Low-density lipoprotein
Desirable Less than 130 mg/dL
Borderline high 140–159 mg/dL
High Greater than 160 mg/dL
Total cholesterol-to-high-density lipoprotein ratio
Desirable Less than 3
Borderline high 3–5
High Greater than 5
Triglycerides
20 years and older Less than 150 mg/dL
Less than 20 years old 35–135 mg/dL

*Values listed are specific for adults or women, if relevant, unless otherwise differentiated. (continued)
56 PROLOG

Normal Values for Laboratory Tests* (continued)

Analyte Conventional Units


Cortisol, plasma
8 am 5–23 micrograms/dL
4 pm 3–15 micrograms/dL
10 pm Less than 50% of 8 am value
Creatinine, serum 0.6–1.2 mg/dL
Dehydroepiandrosterone sulfate 60–340 micrograms/dL
Erythrocyte
Count 3,800,000–5,100,000/mm3
Distribution width 10 plus or minus 1.5%
Sedimentation rate
Wintrobe method 0–15 mm/hour
Westergren method 0–20 mm/hour
Estradiol-17β
Follicular phase 30–100 pg/mL
Ovulatory phase 200–400 pg/mL
Luteal phase 50–140 pg/mL
Child 0.8–56 pg/mL
Ferritin, serum 18–160 micrograms/L
Fibrinogen 150–400 mg/dL
Follicle-stimulating hormone
Premenopause 2.8–17.2 mIU/mL
Midcycle peak 15–35 mIU/mL
Postmenopause 24–170 mIU/mL
Child 0.1–7 mIU/mL
Glucose
Fasting 70–105 mg/dL
2-hour postprandial Less than 120 mg/dL
Random blood 65–110 mg/dL
Hematocrit 36–48%
Hemoglobin 12–16 g/dL
Fetal Less than 1% of total
Hemoglobin A1c (nondiabetic) 5.5–8.5%
Human chorionic gonadotropin 0–5 mIU/mL
Pregnant Greater than 5 mIU/mL
17α-Hydroxyprogesterone
Adult 50–300 ng/dL
Child 32–63 ng/dL
25-Hydroxyvitamin D 10–55 ng/mL
International Normalized Ratio Greater than 1
Prothrombin time 10–13 seconds
Iron, serum 65–165 micrograms/dL
Binding capacity total 240–450 micrograms/dL
Lactate dehydrogenase, serum 313–618 units/L
Leukocytes
Total 5,000–10,000/cubic micrometer
Differential counts
Basophils 0–1%
Eosinophils 1–3%
Lymphocytes 25–33%
Monocytes 3–7%
Myelocytes 0%
Band neutrophils 3–5%
Segmented neutrophils 54–62%

*Values listed are specific for adults or women, if relevant, unless otherwise differentiated. (continued)
Gynecology and Surgery 57

Normal Values for Laboratory Tests* (continued)

Analyte Conventional Units


Lipase
60 years or younger 10–140 units/L
Older than 60 years 18–180 units/L
Luteinizing hormone
Follicular phase 3.6–29.4 mIU/mL
Midcycle peak 58–204 mIU/mL
Postmenopause 35–129 mIU/mL
Child 0.5–10.3 mIU/mL
Magnesium
Adult 1.6–2.6 mg/dL
Child 1.7–2.1 mg/dL
Newborn 1.5–2.2 mg/dL
Mean corpuscular
mCH Hemoglobin 27–33 pg
mCHC Hemoglobin concentration 33–37 g/dL
mCV Volume 80–100 cubic micrometers
Partial thromboplastin time, activated 21–35 seconds
Phosphate, inorganic phosphorus 2.5–4.5 mg/dL
Platelet count 140– 400 × 103 per microliter
Potassium 3.5–5.3 mEq/L
Progesterone
Follicular phase Less than 3 ng/mL
Luteal phase 2.5–28 ng/mL
On oral contraceptives 0.1–0.3 ng/mL
Secretory phase 5–30 ng/mL
Older than 60 years 0–0.2 ng/mL
First trimester 9–47 ng/mL
Second trimester 16.8–146 ng/mL
Third trimester 55–255 ng/mL
Prolactin 0–17 ng/mL
Pregnant 34–386 ng/mL by 3rd trimester
Prothrombin time 10–13 seconds
Reticulocyte count Absolute: 25,000–85,000 cubic micrometers
0.5–2.5% of erythrocytes
Semen analysis, spermatozoa
Antisperm antibody % of sperm binding by immunobead technique;
greater than 20% = decreased fertility
Count Greater than or equal to 20 million/mL
Motility Greater than or equal to 50%
Morphology Greater than or equal to 15% normal forms
Sodium 135–145 mEq/L
Testosterone, female
Total 6–86 ng/dL
Pregnant 3–4 × normal
Postmenopause One half of normal
Free
20–29 years old 0.9–3.2 pg/mL
30–39 years old 0.8–3 pg/mL
40–49 years old 0.6–2.5 pg/mL
50–59 years old 0.3–2.7 pg/mL
Older than 60 years 0.2–2.2 pg/mL
Thyroid-stimulating hormone 0.2–3 microunits/mL
Thyroxine
Serum free 0.9–2.3 ng/dL
Total 1.5–4.5 micrograms/dL

*Values listed are specific for adults or women, if relevant, unless otherwise differentiated. (continued)
58 PROLOG

Normal Values for Laboratory Tests* (continued)

Analyte Conventional Units


Triiodothyronine uptake 25–35%
Urea nitrogen, blood
Adult 7–18 mg/dL
Older than 60 years 8–20 mg/dL
Uric acid, serum 2.6–6 mg/dL
Urinalysis
Epithelial cells 0–3/HPF
Erythrocytes 0–3/HPF
Leukocytes 0–4/HPF
Protein (albumin)
Qualitative None detected
Quantitative 10–100 mg/24 hours
Pregnancy Less than 300 mg/24 hours
Urine specific gravity
Normal hydration and volume 1.005–1.03
Concentrated 1.025–1.03
Diluted 1.001–1.01

*Values listed are specific for adults or women, if relevant, unless otherwise differentiated.
FIG. 10-1

FIG. 46-2

FIG. 46-1
FIG. 53-1

FIG. 65-1.
FIG. 86-1

Acknowledgments
Fig. 2-1, Fig. 6-1, and Fig. 25-1. Images provided by Task Force Co-Chair James M.
Shwayder, MD.
Fig. 10-1. Image courtesy of Gloria A. Bachmann, MD.
Fig. 46-1, Fig. 46-2, and Fig. 53-1. Images were originally published in Edwards L,
Lynch PJ. Genital dermatology atlas. First edition. Philadelphia (PA): Lippincott Williams
& Wilkins; 2004.
Fig. 47-1. Figure is reprinted with permission from “Abdominal wall hernias: imaging
features, complications, and diagnostic pitfalls at multi-detector row CT” by DA Aguirre,
AC Santosa, G Casola, and CB Sirlin. Radiographics 2005;25:1501–20. Copyright 2005,
Radiological Society of North America.
Fig. 86-1. Image provided by Task Force member Colleen K. Stockdale, MD.
Fig. 95-1. Image courtesy of Ghada Mansour, MD.
409 12th Street SW, PO Box 96920
Washington, DC 20090-6920
www.acog.org

You might also like